T03 - Working Capital Finance[1]

March 26, 2018 | Author: Jesha Jotojot | Category: Working Capital, Bonds (Finance), United States Treasury Security, Securities (Finance), Investing


Comments



Description

MANAGEMENT ADVISORY SERVICESWORKING CAPITAL FINANCE WORKING CAPITAL POLICY . Which of the following statements is incorrect about working capital policy? (M) a. A company may hold a relatively large amount of cash if it anticipates uncertain sales levels in the coming year. b. Credit policy has an impact on working capital since it has the potential to influence sales levels and the speed with which cash is collected. c. The cash budget is useful in determining future financing needs. d. Holding minimal levels of inventory can reduce inventory carrying costs and cannot lead to any adverse effects on profitability. e. Managing working capital levels is important to the financial staff since it influences financing decisions and overall profitability of the firm. Brigham Marketable securities Accounts receivable Inventories Net fixed assets Total assets . 2 . As a company becomes more conservative with respect to working capital policy, it would tend to have a(n) a. Increase in the ratio of current liabilities to noncurrent liabilities. b. Decrease in the operating cycle. c. Decrease in the quick ratio. d. Increase in the ratio of current assets to noncurrent assets. CMA 1290 1-23 As a company becomes more conservative in its working capital policy, it would tend to have a(n) A. Decrease in its acid-test ratio. B. Increase in the ratio of current liabilities to noncurrent liabilities. C. Increase in the ratio of current assets to units of output. D. Increase in funds invested in common stock and a decrease in funds invested in marketable securities. CMA 1296 1-8 . Ski Lifts Inc. is a highly seasonal business. The following summary balance sheet provides data for peak and off-peak seasons (in thousands of dollars): Peak Off-peak Cash $ 50 $ 30 RPCPA, AICPA, CMA & CIA EXAMINATION QUESTIONS 20 20 50 500 $620 Spontaneous liabilities $ 30 $ 10 Short-term debt 50 0 Long-term debt 300 300 Common equity 310 310 Total claims $690 $620 From this data we may conclude that (M) a. Ski Lifts has a working capital financing policy of exactly matching asset and liability maturities. b. Ski Lifts’ working capital financing policy is relatively aggressive; that is, the company finances some of its permanent assets with short-term discretionary debt. c. Ski Lifts follows a relatively conservative approach to working capital financing; that is, some of its short-term needs are met by permanent capital. d. Without income statement data, we cannot determine the aggressiveness or conservatism of the company’s working capital financing policy. Brigham Conservative Working Capital Policy *. Compared to other firms in the industry, a company that maintains a conservative working capital policy will tend to have a (D) a. Greater percentage of short-term financing. b. Greater risk of needing to sell current assets to repay debt. c. Higher ratio of current assets to fixed assets. d. Higher total asset turnover. RPCPA 0595 1 0 40 100 500 $690 Aggressive Working Capital Policy *. A firm following an aggressive working capital strategy would (M) a. Hold substantial amount of fixed assets. b. Minimize the amount of short-term borrowing. c. Finance fluctuating assets with long-term financing. d. Minimize the amount of funds held in very liquid assets. RPCPA 1091 3 . The working capital financing policy that subjects the firm to the greatest risk of being unable to meet the firm’s maturing obligations is the policy that finances (E) a. Fluctuating current assets with long-term debt. b. Permanent current assets with long-term debt. c. Permanent current assets with short-term debt. d. Fluctuating current assets with short-term debt. CMA 1295 1-2 4 . Since Marsh, Inc. is experiencing a sharp increase in sales activity and a steady increase in production, the management of Marsh has adopted an aggressive working capital policy. Therefore, the company's current level of net working capital Page 1 of 39 MANAGEMENT ADVISORY SERVICES WORKING CAPITAL FINANCE A. Would most likely be the same as in any other type of business condition as business cycles tend to balance out over time. B. Would most likely be lower than under other business conditions in order that the company can maximize profits while minimizing working capital investment. C. Would most likely be higher than under other business conditions so that there will be sufficient funds to replenish assets. D. Would most likely be higher than under other business conditions as the company's profits are increasing. CMA 0689 1-11 Aggressive vs. Conservative Working Capital Policy 5 . Clay Corporation follows an aggressive financing policy in its working capital management while Lott Corporation follows a conservative financing policy. Which one of the following statements is correct? A. Clay has a low ratio of short-term debt to total debt while Lott has a high ratio of shortterm debt to total debt. B. Clay has a low current ratio while Lott has a high current ratio. C. Clay has less liquidity risk while Lott has more liquidity risk. D. Clay's interest charges are lower than Lott's interest charges. CMA 1284 1-22 RPCPA, AICPA, CMA & CIA EXAMINATION QUESTIONS Page 2 of 39 MANAGEMENT ADVISORY SERVICES WORKING CAPITAL FINANCE WORKING CAPITAL MANAGEMENT Appropriate Level of Working Capital 6 . Determining the appropriate level of working capital for a firm requires (E) a. Evaluating the risks associated with various levels of fixed assets and the types of debt used to finance these assets. b. Changing the capital structure and dividend policy for the firm. c. Maintaining short-term debt at the lowest possible level because it is ordinarily more expensive than long term debt. d. Offsetting the profitability of current assets and current liabilities against the probability of technical insolvency. e. Maintaining a high proportion of liquid assets to total assets in order to maximize the return on total investments. CMA 0689 1-10, 0696 1-16, RPCPA 0596 7 . Which of the following statements about current asset management is most correct? (E) a. A positive net float means that a company has more cash available for its use than the amount shown in the company’s books. b. Use of a lockbox reduces the possibility that petty cash will be lost. c. Depreciation has an impact on the cash budget. d. Statements a and c are correct. Brigham Working Capital Working Capital Defined *. The amount of long-term capital that is made to revolve in conducting operations and serves as the lifeblood of the company (E) a. Paid-up capital c. Working capital b. Net worth d. None of these RPCPA 0588 8 . The working capital is the difference between A. Current assets and current liabilities. C. Total assets and total liabilities. B. Fixed assets and fixed liabilities. D. Equity and cash. CMA 0692 1-25 3. Net Working Capital is the: A. Difference between short-term assets and short term liabilities B. Difference between long-term assets and long term liabilities C. Difference between long-term assets and short term liabilities D. None of the above RPCPA, AICPA, CMA & CIA EXAMINATION QUESTIONS B&M Effect of Transactions on Working Capital 9 . Starrs Company has current assets of $300,000 and current liabilities of $200,000. Starrs could increase its working capital by the (E) A. Prepayment of $50,000 of next year's rent. B. Refinancing of $50,000 of short-term debt with long-term debt. C. Purchase of $50,000 of temporary investments for cash. D. Collection of $50,000 of accounts receivable. CMA 1293 1-19 10 . Starrs has current assets of 300,000 and current liabilities of 200,000. Starrs could increase its working capital by the A. Prepayment of 50,000 of next year's rent. B. Refinancing of 50,000 of short-term debt with long-term debt. C. Acquisition of land valued at 50,000 through the issuance of common shares. D. Purchase of 50,000 of financial assets held for trading for cash. CMA 1293 1-19 *. Which of the following transactions causes an increase in working capital? a. Sale of merchandise on credit at a price above cost. b. Sale of marketable securities at a price below cost. c. Collection of an account receivable. d. Return to supplier of defective merchandise purchased on credit. Full credit allowed by supplier. RPCPA 1075 . Other things held constant, which of the following will cause an increase in working capital? (M) a. Cash is used to buy marketable securities. b. A cash dividend is declared and paid. c. Merchandise is sold at a profit, but the sale is on credit. d. Long-term bonds are retired with the proceeds of a preferred stock issue. e. Missing inventory is written off against retained earnings. Brigham 11 . Which one of the following would increase the working capital of a firm? A. Cash payment of payroll taxes payable. B. Purchase of a new plant financed by a 20-year mortgage. C. Cash collection of accounts receivable. D. Refinancing a short-term note payable with a two-year note payable. 12 CMA 1294 1-15 . If a firm increases its cash balance by issuing additional shares of common stock, working capital (E) a. Remains unchanged and the current ratio remains unchanged. Page 3 of 39 Minimizing taxes. Speculative. Increases and the current ratio decreases. this will reduce its customers’ net float. Determining the appropriate target cash balance involves assessing the trade-off between: A. In the early 1980s. CIA 0593 IV-28 Comprehensive 14 . Transactional. CMA 1295 I-12 1. b. Speculative. d. and speculative purposes. Income and diversification B. the prime interest rate hit a high of 21 percent. Cash surplus investment CMA 1294 1-30 13 . Maximizing rate of return. I only. A precautionary motive for holding excess cash is (E) a. c. WORKING CAPITAL FINANCE a. CMA & CIA EXAMINATION QUESTIONS Page 4 of 39 . RPCPA 0595 Which of the following is not a major function in cash management? (E) RPCPA. d. I and III only. The following transactions occurred during a company's first year of operations: I. AICPA. c. Borrowed money by issuance of short-term debt III. Liquidity and safety. Transactional. Maximizing sales d. Increases and the current ratio remains unchanged. To enable a company to meet the cash demands from the normal flow of business activity. B. c. d. b. psychological. To enable a company to avail itself of a special inventory purchase before prices rise to higher levels. this will increase its net float and thus reduce its required cash balances. To avoid having to use the various types of lending arrangements available to cover projected cash deficits. D. Increases and the current ratio increases. Which of the following statements is most correct? (E) a. All of the following statements in regard to working capital are correct except (M) a. Investing in common stock due to the dividend exclusion for federal income tax purposes. social. Profitability varies inversely wit liquidity. Transaction-related needs are less than cash inflows C. Financing permanent inventory buildup with long-term debt is an example of an aggressive working capital policy. e. I and II only. d. fiduciary. That sharp interest rate decline has increased firms’ concerns about the efficiency of their cash management programs. c. A firm that has such an efficient cash management system that it has positive net float can have a negative checkbook balance at most times and still not have its checks bounce. Purchased treasury stock Which of the items above caused a change in the amount of working capital? A. and transactional purposes. If a firm can get its customers to permit it to pay by wire transfers rather than having to write checks. The benefit and cost of liquidity C. CMA 1286 1-32 *. c. To enable a company to have cash to meet emergencies that may arise periodically. Cash flow control b.MANAGEMENT ADVISORY SERVICES b. b. Obtaining financing services RPCPA 0590 2. CMA 0696 1-29 CASH MANAGEMENT Function 15 . If a firm begins to use a well-designed lockbox system. c. A good cash management system would minimize disbursement float and maximize collections float. According to John Maynard Keynes. C. When managing cash and short-term investments. D. In 2000 the prime rate was considerably lower. e. Purchased a delivery van for cash I. Firms would need to hold zero cash when: A. precautionary. All of the above B&M *. C. II and III only. b. Transaction-related needs are greater than cash inflows B. Investing in Treasury bonds since they have no default risk. B. Balance sheet strength and transaction needs D. The hedging approach to financing involves matching maturities of debt with specific financing needs. and precautionary purposes. Brigham Motives for Holding Cash 17 . and social purposes. a corporate treasurer is primarily concerned with (E) a. Current liabilities are an important source of financing for many small firms. Transaction-related needs are not perfectly synchronized with cash inflows D. d. Transaction-related needs are perfectly synchronized with cash inflows B&M 16 . the three major motives for holding cash are for A. Uncertainty about day-to-day or week-to-week cash flows decreases B&M Methods of Accelerating Cash Collections 32. Running lock-boxes B&M C. We should expect cash balances to increase when: A. Meet future needs. Disbursement float B. Which of the following statements is most correct? (E) a. Precautionary balance. Availability float B. They are set up to handle disbursement activity B. The master and the zero balance account locate at the same bank B&M 26. CMA & CIA EXAMINATION QUESTIONS B&M 13. Providing advice and references D. b. Maintain adequate cash needed for transactions. A cash management system that maximizes collections float and minimizes disbursement float is better than one with lower collections float and higher disbursement float. C. AICPA.MANAGEMENT ADVISORY SERVICES WORKING CAPITAL FINANCE 18 . A large firm may hold substantial cash balances because: A. b. Checks are automatically transferred into the account as checks presented for payment D. A cash management system that minimizes collections float and maximizes disbursement float is better than one with higher collections float and lower disbursement float. compensating. Compensating balance. The difference between bank cash and book cash is called: A. Corporations have few bank accounts and it is difficult to manage their cash D. precautionary. B. All of the above Disbursement Float 15. The use of a lockbox is designed to minimize cash theft losses. None of the above B&M Maximizing Net Float 20 . CMA 0694 1-22 . The total desired cash balance can be determined by calculating the amount needed for each purpose and then summing them together. The company may have a very decentralized organization D. Checks written by the firm are said to generate: A. The easier a firm’s access to borrowed funds the higher its precautionary balances will be. The account has a minimum amount at all times C. Net float D. Banks generally offer the following services: A. For some firms. Processing checks C. The cash balances of most firms consist of transactions. Which of the following statements is most correct? (M) a. in order to protect against sudden increases in interest rates. All of the following are valid reasons for a business to hold cash and marketable securities except to A. The amount of cash that a firm keeps on hand in order to take advantage of any bargain purchases that may arise is referred to as its A. B. Availability float C. c. Brigham d. then the lockbox should be installed. Sales volume falls D. Book float B&M 16. holding highly liquid marketable securities is a substitute for holding cash because the marketable securities accomplish the same objective as cash. Disbursement float C. Concentration banking C. D. Banks generally offer the following cash management services: A. Ledger float D. All of the above RPCPA. 12. Cash is costless and need not be managed closely B&M 14. c. Earn maximum returns on investment assets. Running lock-boxes B. Interest rates increase C. Speculative balance. Which of the following is used to control disbursements? A. Brigham Page 5 of 39 . CIA 1194 IV-15 25. Lock-box system B. Satisfy compensating balance requirements. Zero-balance account D. Transferring funds D. It is too difficult to estimate the costs of security transactions B. All of the above B&M 33. The company may have accounts in many different banks C. C. These balances are required by the bank B. D. and speculative balances. Processing checks B. The transfer is automatic and involves an accounting entry only E. Banks are compensated by account balances for payment of services C. Firms today are more likely to rely on cash than on reserve borrowing power or marketable securities for speculative purposes because of the need to move quickly. If the cost of the lockbox is less than theft losses saved. Transactions balance. Which of the following statements concerning zero balance accounts is not correct? A. Most large firms hold a cash balance greater than most models imply because: A. Fedwire B&M 19 . The transaction costs of buying or selling interest-bearing securities increase B. A draft. A check processed through ACH (Automated Clearing House): A. C. including: A. A working capital technique that increases the payable float and therefore delays the outflow of cash is (E) A. . Disbursement float B&M 24 . b. A lock-box system A. Electronic cash transfers offer several advantages. Reduces the risk of having checks lost in the mail. An automated clearing house (ACH) electronic transfer is a(n) a. lock-boxes can be used to reduce: A. None of the above B&M Lock-Box System 21. Check that must be immediately cleared by the Federal Reserve Bank. By getting closer to the source of payment. Factoring. Which of the following statements is most correct? (M) a.MANAGEMENT ADVISORY SERVICES WORKING CAPITAL FINANCE d. b. Reduces the need for compensating balances. Concentration banking. d. Easy automation of record-keeping B. A low marginal transactions cost C. c. Will take two or three days to clear B. C. . In-house processing B&M 23. In-house processing float B. Make collections over a wide geographic area. C. Lockbox systems are used mainly for security purposes as well as to decrease the firm’s net float. Electronic payment to a company’s account at a concentration bank. Availability or clearing float C. Computer-generated deposit ticket verifying deposit of funds. B. e. A firm practicing good cash management and making use of positive net float will bring its check book balance as close to zero as possible. A lockbox system. Check-like instrument drawn against the payor and not against the bank. Brigham Draft 22 . Electronic funds transfer. Mail float D. A reduced float D. Lock-boxes B. but must never generate a negative book balance. Hold inventories at many different sites. If a firm can speed up its collections and slow down its disbursements. c. AICPA. a firm will need a smaller line of credit if it can arrange to pay its bills by the 5th of each month than if its bills come due uniformly during the month. Accelerates the inflow of funds. RPCPA. Have widely disbursed manufacturing facilities. Concentration banking C. d. All of the above B&M 31. Will clear immediately C. B. A lock-box system. Electronic Data Interchange (EDI). Other things held constant. Will clear the same day D. Have a large marketable securities account to protect. 21 . Wire transfers D. CMA 0694 1-23) d. A working capital technique that delays the outflow of cash is (E) A. CMA 1293 1-21 23 . D. b. The most common cash management technique used to speed up collections is: A. CMA & CIA EXAMINATION QUESTIONS CMA 1293 1-20 Brigham Page 6 of 39 . D. D. it will be able to reduce its net float. CMA 1296 1-5 Automated Clearing House (ACH) Electronic Transfer 27. B. A draft. c. e. Provides security for late night deposits. Poor synchronization of cash flows that results in high cash management costs can be partially offset by increasing disbursement float and decreasing collections float. Send payables over a wide geographic area. A lockbox plan is most beneficial to firms that (M) a. The size of a firm’s net float is primarily a function of its natural cash flow synchronization and how it clears its checks. c. The fastest but most expensive way to transfer surplus funds from the local deposit bank to the concentration bank is: A. All of the above are correct B&M 11. Maintaining compensating balances rather than paying cash for bank services. D. WORKING CAPITAL FINANCE The following practices will impact the cash flow of the company: (E) a. Minimizing total costs less trading costs B&M 10. Sales personnel are unequivocally responsible for collecting their credit sales. The Baumol model determines the optimal cash balance by: A. The optimum cash balance is higher at higher interest rates B. which will typically be sold to other firms on credit. Methods of accelerating cash collections include all of the following except (E) A. Minimize the use of float. Baumol's model of cash balances states that: Q = [(2 x T x C. All of the above B&M Comprehensive *. Lockbox systems. Minimizing total costs of holding cash against trading securities costs C. Having a safety stock set to zero D. B. The Baumol cash balance model is limited by: A. Which of the following is a way for companies to speed up collections? A. CMA 0689 1-12 RPCPA. The optimum cash balance is higher at higher fixed order costs C.5 where T = annual cash disbursement. B./i]^0. Assuming the cash flows are variable across the period B. b. Balancing total costs against opportunity costs B. AICPA. Balancing trading securities costs against total costs D. c. Wire Transfer 28. Electronic funds transfers. Increased synchronization of cash flows. RPCPA 0592 25 . Which of the following actions would not be consistent with good management? (M) a. Reducing unit costs of production. A mail float system E. Concerning the Baumol model. b. Sales commissions are based on collected invoices. The minimum holding of cash B&M 9. which of the following is not correct (all other things equal)? A. The average holding of bills C. Page 7 of 39 . Decentralized collections. The number of times per annum bill should be sold B. Maintaining an average cash balance equal to that required as a compensating balance or that which minimizes total cost. Establishing multiple collection centers throughout the country. d. Remote disbursing C. A typical firm doing business nationally cannot expect to accelerate its cash inflow by A. which it wishes to reduce over the budget period of one year. What is Q? A. Taking longer credit from suppliers without any loss of discounts. c. RPCPA 0594 d. A lock-box system D. C. C = cost per sale of T-bills. A company’s management is concerned about the large bank overdraft. Use of checks and drafts in disbursing funds. d. Baumol model B. C. The cash conversion cycle resulting from this new product can be measured as the length of time from A. Reducing wastage and loss through damage of regularly-used stock items. b. CMA 1283 1-23 D. A smooth disbursement rate and now cash inflows over the period C. Concentration banking D.MANAGEMENT ADVISORY SERVICES *. The amount of T-bills that should be sold at any one time D. Statement of accounts receivable are reconciled with customers and regularly sent for confirmation. i = interest rate. CMA & CIA EXAMINATION QUESTIONS 26 . Both B and C B&M Cash Conversion Cycle 27 . The optimum cash balance is higher at higher total cash requirement D. Employing a lockbox arrangement. Which one of the four items below will not necessarily result in a lower bank overdraft? (E) a. An availability float system C. Automatic transfer of funds is arranged with banks regarding deposits of branches. Compensating balances. An in-house processing float system B. Initiating controls to accelerate the deposit and collection of large checks. Cash purchases of raw materials to the collection of accounts receivable. Reducing bad debts. An enterprise plans to produce a new product. A wire transfer B&M Concentration Banking 22. Baumol’s Model of Cash Balances 3. RPCPA 0595 *. Statements a and b are correct. its decision will be based upon A. Maintain the level of receivables as sales decrease. Cash purchases of raw materials to the sale of the product. When managing cash and short-term investments. which of the following measures would tend to reduce the cash conversion cycle? (M) a. Investing in Treasury bonds since they have no default risk. Liquidity and safety.” They then proceed to use cash management module. Statements a and b are correct. When a company is evaluating whether the ratio of cash and marketable securities to total assets should be high or low. Brigham Optimal Mix Between Cash and Short-term Investments 30 . The economic order quantity (EOQ) formula can be adapted in order for a firm to determine the optimal mix between cash and marketable securities. B. The company starts paying its bills sooner. Post-dated checks are not deposited on time upon maturity. Adopting a just-in-time inventory system that reduces the inventory conversion period. In terms of the rate of return forgone on converted securities and the cost of such transactions. Marketable securities level. Which of the following steps would reduce its cash conversion cycle? (E) a. CMA 0689 1-15 Comprehensive *. CMA 1295 1-12 31 . Determining the amount and timing of conversions of marketable securities to cash is a critical element of a financial manager's performance. An opportunity cost is associated with holding cash. b. Reducing the average days sales outstanding (DSO) on its accounts receivable. such as the EOQ model to determine the a. d. CIA 0593 IV-52 34 . C. c. The company increases its average inventory without increasing its sales. CMA & CIA EXAMINATION QUESTIONS . RPCPA. Reducing the amount of time the company takes to pay its suppliers. d. Maximizing rate of return. c. Minimizing taxes. Most sales are on cash basis and receivables are aged “current” c. Brigham 29 . e. Inversely related to the rate of return forgone and inversely related to the cost of the transaction. Page 8 of 39 . the optimal amount of cash to be raised by selling securities is A. B. Directly related to the rate of return forgone and directly related to the cost of the transaction. b.MANAGEMENT ADVISORY SERVICES WORKING CAPITAL FINANCE B. Forgo discounts that are currently being taken. Cash is collected at the earliest time possible. The cost of a transaction is independent of the dollar amount of the transaction and interest rates are constant over the short run. b. d. C. When the product is completed to the sale of the product. Cash purchases of raw materials to the time the final product is completed. B. Cash flow requirements are random. C. Helena Furnishings wants to sharply reduce its cash conversion cycle. Risk-profitability trade-off considerations. CMA 1286 1-33 33 . D. Take discounts when offered. Buy more raw materials to take advantage of price breaks. a corporate treasurer is primarily concerned with (E) A. The EOQ model assumes all of the following except a. which reduces its average accounts payable without reducing its sales. c. beginning with the first dollar. b. D. Financial leverage considerations. Proper relationship between current assets and current liabilities. AICPA. Brigham . The total demand for cash is known with certainty. Flotation cost considerations. Directly related to the rate of return forgone and inversely related to the cost of the transaction. CMA 0684 1-5 32 28 . Some managers express the opinion that “cash management problems are nothing more than inventory problems. c. d. D. Ignoring cost and other effects on the firm. Credit and collection policies. b. Which of the following actions are likely to reduce the length of a company’s cash conversion cycle? (M) a. D. Inversely related to the rate of return forgone and directly related to the cost of the transaction. The company reduces its DSO. Offer a longer deferral period to customers. b. Operating leverage considerations. The following are desirable in cash management except: (E) a. c. Proper blend of marketable securities and cash. CIA 1193 IV-52 C. d. and payable deferral periods. The criteria that should be considered in investing surplus cash (E) a. AICPA. B. payment. The firm has just sold long-term securities and has not yet invested the proceeds in earning assets. Commercial paper. federal agency discount notes. Funds sourcing and custodianship must be done at the lowest possible cost. T-bills C. All sales are properly receipted and promptly deposited intact. Brigham Money Market Instruments 7. taxability. The operating. Treasury bills.MANAGEMENT ADVISORY SERVICES d. All of the above RPCPA 0588 35 . and default costs are costs associated with cash management. WORKING CAPITAL FINANCE RPCPA 0593 In cash management. c. Obtaining financing services and controlling cash flow are some of the major functions of cash management. b. Yield on the principal b. Capital costs. RPCPA 1090 MARKETABLE SECURITIES Criteria *. The firm must meet a known financial commitment. Which of the following statement completions is most correct? If the yield curve is upward sloping. Preferred stocks B&M 8. Default risk. c. T-bills C. medium-term notes B&M Marketable Securities Portfolio . Inventory conversion. Weighted toward short-term securities to avoid interest rate risk. Safety of the company’s funds c. b. and interaction with capital markets. such as financing an ongoing construction project. and cash conversion cycles. which of the following statements is false? (M) a. where excess funds must be invested for a return that is best in the market. assumed to be held for liquidity purposes. Short costs. Brigham RPCPA. then a firm’s marketable securities portfolio. bankers' acceptances. Liquidity of the principal d. long costs. S. C. should be (M) a. major investment decisions. The following are money market instruments except: A. Commercial paper B. b. *. Negotiable CDs. Federal agency discount notes. CIA 0589 IV-52 D. Which of the following are criteria for selection among securities available for a marketable securities portfolio? (M) A. Weighted toward long-term securities because they pay higher rates. and relative yields. The firm has purchased a fixed asset that will require a large write-off of depreciable expense. commercial paper. and procurement costs are costs associated with optimal cash balance model approach c. d. d. repurchase agreements. repurchase agreements B. The firm must finance seasonal operations. delinquency costs.and short-term securities to minimize the effects of either an upward or a downward trend in interest rates. Federal agency discount notes D. The three money market securities with the greatest volume of business are: A. tax-exempt municipal notes D. d. e. Treasury securities to avoid interest rate risk. . receivables conversion. Balanced between long. Weighted toward U. CMA & CIA EXAMINATION QUESTIONS Page 9 of 39 . Weighted toward short-term securities because they pay higher rates. Which of the following is not a situation that might lead a firm to hold marketable securities? (M) a. Planning. Negotiable commercial papers C. CMA & CIA EXAMINATION QUESTIONS 10. Which of the following financial instruments can be traded in international money markets? A. B. Are regulated by the Federal Reserve System. Have yields considerably greater than bankers' acceptances and commercial paper. Commercial paper D. B. Common stock B. C. Offers a highly competitive trading market B&M 17. Treasury bills. corporate bonds c. US Government B. a. Aaa corporate bonds. The marketable securities with the least amount of default risk are A. All of the above B&M Page 10 of 39 . in advance. commercial paper b. Treasury securities. The commercial paper of a Aaa rated company. Treasury bonds e. None of the above B&M CMA 0688 1-15 B&M Floating Rate Preferred Stock 16. Repurchase agreements B&M 37 . Has a fixed rate of dividend income D. which may reduce the after-tax value of the instrument D. Even though the dividend rate on a floating-rate preferred stock is floating to keep in line with interest rates. CIA 1195 IV-65 Repurchase Agreement 15. C. Have a secondary market for investors. Negotiable CDs are issued by: A. Which security is most often held as a substitute for cash? (E) A.000. Gold. An investor buys part of a government security dealer's inventory and simultaneously agrees to sell it back D. Preferred shares. The impact of tax law changes. the instrument still suffers from risk such as: A. CMA 0691 1-11 38 Of the following four investments. Still provides the corporate investor with the tax exclusion on dividend income C. D. C. Municipal bonds. B. Are usually sold in denominations of a minimum of $100. Stock options of a Aaa rated company. D. Repurchase agreements.S. AICPA. Is not rated by Moody's or Standard & Poor's B. Treasury bills is considered to be the safest. U. For which of the following investments is there a very active secondary market? A. Preferred stock C. Medium-term notes C. to re-invest money in a negotiable certificate of deposit C. CMA 0691 1-12 D. EuroCommercial Paper 14. C. C. Federal agencies C. The best example of a marketable security with minimal risk would be (D) A. B. A bank depositor agrees. US Treasury bills B. Commercial paper. Long-term bonds B&M 21. The common stock of a Aaa rated company. Floating-rate preferred stock offers competitive rates of return with traditional money-market instruments but: A. Agency issues 39 . Common stock.S. Corporations Commercial Paper 41 . (E) d. U. Banks D. A thin market causing potential principal risk and liquidity concerns B.MANAGEMENT ADVISORY SERVICES WORKING CAPITAL FINANCE Treasury Bills 36 .S. D. Treasury bonds. A company agrees to buy back its commercial paper before maturity B. Treasury bills. The federal government agrees to buy T-bills B&M Negotiable Certificates of Deposit 40 . RPCPA. MTNs B. A tax-paying corporation would prefer to invest short-term money in: A. Mortgages. Commercial papers sold in the international markets are called: A. Federal government agency securities. Floating-rate preferred stock D. Which one of the following is not a characteristic of a negotiable certificate of deposit? Negotiable certificates of deposit (D) A. U. A repurchase agreement occurs when: A. S. EuroCommercial papers D. D. The risk of downgrades from the narrow range of issuers C. B. U. CMA 1289 1-14 9. None of the above B&M 43 . Dollar deposits in banks outside the US B.S. Page 11 of 39 . Has interest payments linked to the level of short-term interest rates C. B. Which of the following investments is not likely to be a proper investment for temporary idle cash? (E) a. Treasury bills. Deutsche dollars C. Treasury bonds due within one year. Betting that a stock will increase by a certain amount within a given period of time. d. In smaller businesses in which the management of cash is but one of numerous functions performed by the treasurer. Selling of a security that is not owned by the seller. American depositing receipts D. Eurodollars. B. Always sold in some country other than the one in whose currency the bond is denominated. Commercial paper. Eurodollar deposits are outside the direct control of the U. The deposits involve different currencies. B. C. Generally issued as registered bonds. RPCPA. c. Interest rates on Eurodollar deposits are tied to the London Interbank Offer Rate (LIBOR). CIA 0595 IV-59 3. C. Convertible bonds. Eurodollar deposits are U. various cost incentives and diversification arguments suggest that surplus cash should be invested in A. b. May periodically be sold back to the issuer at face value D. Selling of all the shares you own in a company in anticipation that the price will decline dramatically. Commercial paper. Both A and B D. Eurodollars B. All of the above B&M Not a Proper Investment for Idle Cash 46 . AICPA. C. The term “short selling” is the a. Bankers' acceptances. There is a difference between "Eurodollar" or "international dollar" rate and short-term interest rate in the US because: A. rates on equivalent instruments. monetary authorities and has lower costs. b.S. CMA & CIA EXAMINATION QUESTIONS D. Which of the following statements does not properly describe a Eurodollar deposit? A. Of the following. Dollars held by foreign governments D. c. None of the above B&M Variable Rate Demand Bond 11. Banks are not subject to reserve requirements on international dollars B. Commercial paper. All of the following are alternative marketable securities suitable for investment except A. U. C. US dollars deposited in a German bank are called: A. dollar deposits in banks outside of the U. D. rates on equivalent instruments.S.S. "Eurodollars" or "international dollars" are: A.MANAGEMENT ADVISORY SERVICES WORKING CAPITAL FINANCE Money Market Mutual Funds 42 . rates on equivalent instruments because A. 45 . Corporate bonds. The Eurodollar market is outside the direct control of the U. a characteristic of Eurobonds is that they are A.S. None of the above B&M 44. Always denominated in Eurodollars. Money market mutual funds. B. Eurodollar deposit rates tend to be lower than domestic U. Dollars deposited in the US by foreigners C.S. D. CIA 1195 IV-66 44 .S. Interest rates received by depositors on Eurodollar deposits tend to be higher than domestic U. CMA 0694 1-25 *. CMA 1280 1-1 Eurodollar Deposit 2. CMA 1294 1-16 d. Treasury bills. RPCPA 0595 Short Selling 47 . Eurodollar deposits are for smaller amounts. CIA 0594 IV-58 D. C. Is a long-term security B. Initial public offering of an established profitable conglomerate. Borrowers pay higher rates than domestic U. D. B. Is tax-exempt E. Sold outside the country of the borrower but are denominated in the currency of the country in which the issue is sold. Selling of a security that was purchased by borrowing money from a broker.S. monetary authorities. Banks need not insure international dollar deposits with the FDIC C. A variable rate demand bond (VRDB): A. c. they can provide erroneous signals to credit managers when (E) a. then its aging schedule would probably show some past due accounts. then the firm’s DSO will probably increase. Average length of time that receivables have been outstanding. net 30. Aging schedules can be constructed from the summary data provided in the firm’s financial statements. Brigham Aging of Accounts Receivable 50 . Some customers take the discount and others do not. If the firm maintains stable growth it will also be able to maintain its accounts receivable at its current level. b. Purchasing only the longest maturities for given rates of return. d. The inventory conversion period. In managing a firm’s accounts receivable it is possible to increase credit sales per day yet still keep accounts receivable fairly steady if the firm can shorten the length of its collection period. c. d. d. Percentage of sales that have been collected after a given period of time. The goal of credit policy is to (E) a. Adherence to the liquidity preference theory of securities investment. d. b. Swapping different maturities of similar quality debt securities in order to obtain higher yield. The inventory divided by average daily sales. B. The DSO of a firm with seasonal sales can vary. b. Other things held constant. If a firm sells on terms of 2/10. Amount of receivables that have been outstanding for given lengths of time. A firm that makes 90 percent of its sales on credit and 10 percent for cash is growing at a rate of 10 percent annually. AICPA. Since receivables and payables both result from sales transactions. Sales fluctuate seasonally. If a firm that sells on terms of net 30 changes its policy and begins offering all customers terms of 2/10. Page 12 of 39 . the impact on sales can’t be determined because the increase in the discount is offset by the longer net terms. Extend credit to the point where marginal profits equal marginal costs. b. An enterprise's receivables collection period is equal to A. the better its credit department. d. If a firm changes its credit terms from 1/20. to 2/10. the accounts receivable balance will be high or low depending on the season. Before a typical account becomes delinquent. C. After a typical credit sale is made until the firm receives the payment. net 40 days. b. net 30. CMA 1290 1-22 c. d. Which of the following statements is most correct? (M) a. c. If a firm’s volume of credit sales declines then its DSO will also decline. c. However. Which of the following statements is most correct? (M) a. Beyond the end of the credit period before a typical customer payment is received. In the process of investing of surplus cash. d. The cash conversion cycle. which tends to reduce sales. CMA & CIA EXAMINATION QUESTIONS CIA 0597 IV-27 49 . The day's sales outstanding. If a firm has a large percentage of accounts over 30 days old. a firm with a high receivables-to-sales ratio should also have a high payables-to-sales ratio. net 60 days. RPCPA. An aging schedule is used to determine what portion of customers pay cash and what portion buy on credit. either seasonally or cyclically. it is a sign that the firm’s receivables management needs to be reviewed and improved. the higher a firm’s days sales outstanding (DSO). Brigham Monitoring Receivables . and its DSO is 30 days. b. An aging of accounts receivable measures the a. . Brigham Credit & Collection Policy *. since the 10 percent cash sales can be used to manage the 10 percent growth rate.MANAGEMENT ADVISORY SERVICES WORKING CAPITAL FINANCE Riding the Yield Curve *. Brigham Days Sales Outstanding 48 . c. D. Customers’ payments patterns are changing. Which of the following statements is most correct? (M) a. and if no change in sales volume occurs. It takes a typical check to “clear” through the banking system. CMA 1295 1-4 . c. Sales are relatively constant. While the sales per day figure is usually based on the total annual sales. Ability of the firm to meet short-term obligations. Statements a and c are correct. b. Analyzing days sales outstanding (DSO) and the aging schedule are two common methods for monitoring receivables. Diversifying securities portfolio so that the firm has an equal balance of long-term versus short-term securities. e. RPCPA 0596 RECEIVABLES MANAGEMENT . The average collection period for a firm measures the number of days (E) a. the term “riding the yield curve” refers to (E) a. *. Sales have gone up. Receivable Turnover 54 . means that if the original sale took place on February 1st. Quality of accounts accepted. CMA 1296 1-18 . The orgnization can borrow funds at a rate less than the annual interest cost. Inventories have gone up. c. Days’ sales in receivables x accounts receivable turnover. Seasonal dating with terms 2/15. The minimum risk group to which credit should be extended. A significant sales volume decrease near the end of the accounting period. the company is attempting to (M) A. Average daily sales x average collection period. It normally stimulates sales. Increase the average collection period. Which of the following represents a firm’s average gross receivables balances? I. could indicate that the firm A. I and II only. and the volume of credit sales also increases. which implies a very short days-sales outstanding. then the firm’s accounts receivable will automatically increase. b. The write-off of an uncollectible account (assume the use of the allowance for doubtful accounts method). D. It is possible for a firm to overstate profits by offering very lenient credit terms that encourage additional sales to financially “weak” firms. d. c. then why do firms take such actions? (E) a. Accounts receivable turnover will normally decrease as a result of (E) a. The extent (in terms of money) to which a firm will go to collect an account. D. Firms use seasonal dating primarily to decrease their DSO. c. b.MANAGEMENT ADVISORY SERVICES WORKING CAPITAL FINANCE RPCPA 0597 e. To increase the firm’s deferral period for payables. A major disadvantage of such a policy is that it is likely to increase uncollectible accounts. If easing a firm’s credit policy lengthens the collection period and results in a worsening of the aging schedule. It is collecting credit sales more slowly than before. RPCPA. and has short credit terms. II only. (E) a. B. RPCPA 1095 Average Gross Receivables Balance 53 . Level of collection expenditures. If credit sales as a percentage of a firm’s total sales increases. Page 13 of 39 . CMA 0684 1-3 B. Gleim b. the customer can take the discount up until March 15th. A decrease in the firm's receivable turnover ratio means that (D) A. b. but must pay the net invoice amount by April 1st. Quantity discount given. Cash discount given. A high turnover of accounts receivable. D. 51 52 . II and III only. Has a relaxed (lenient) credit policy. To meet competitive pressures. Increase bad-debt losses. A change in credit policy to lengthen the period for cash discounts. Minimize bad debt losses. c. Brigham Which of the following statements is most correct? (M) a. C. d. Offers small discounts. Which one of the following items is not considered as part of the firm’s credit policies? (M) a. Net sales ÷average gross receivables. B. C. Maximize profits. Has an inefficient credit and collection department. A firm with excess production capacity and relatively low variable costs would not be inclined to extend more liberal credit terms to its customers than a firm with similar costs that is operating close to capacity. B. c. CMA & CIA EXAMINATION QUESTIONS 55 . The one item listed below that would warrant the least amount of consideration in credit and collection policy decisions is the A. b. c. b. with April 1 dating. The length of time for which credit is extended. Uses a lockbox system. D. Maximize sales. When a company analyzes credit applicants and increases the quality of the accounts rejected. The organization can borrow funds at a rate exceeding the annual interest cost. I only. AICPA. net 30 days. An organization would usually offer credit terms of 2/10. The size of the discount that will be offered. C. C. d. Gleim 56 . CMA 1286 1-34 . II. An increase in cash sales in proportion to credit sales. CMA 0690 4-14 Credit Terms 57 . . III. synchronizes cash flows. b. It is collecting credit sales more quickly than before. d. d. Statements a and b are correct. Minimize collection expenses. Brigham It is held that the level of accounts receivable that the firm has or holds reflects both the volume of a firm’s sales on account and a firm’s credit policies. net 30 when a. The cost of capital approaches the prime rate. d. Maximize sales. c. d. we know that: (E) a. Increase in the level of receivables only. A decrease in the cash conversion cycle. credit limits remain the same. sales would increase and. c. and credit terms remain the same. Most competitors are offering the same terms and the organization has a shortage of cash. The sales manager at Ryan Company feels confident that. b. an increase in discounts taken. consequently. and aging of the receivables is improving. An increase in the average collection period. d. B. Net profit has increased. b. CMA 1289 1-15 *. Average collection period has decreased. A change in credit policy has caused an increase in sales. net/30 basis. c.000 $600. d. The size of the discount offered has decreased. Decreased receivables. c. B. Accounts receivable turnover has decreased. current receivables ratio to past due increases. and a reduction in the number of doubtful accounts. CMA 1296 1-13 Change in Credit Policy 59 . Percentage discount offered has decreased. if the credit policy at Ryan’s were changed. the company would utilize excess capacity. The likely effect of this move is (M) a. If a firm had been extending trade credit on a 2/10. A change in credit policy has caused an increase in sales. RPCPA 0596 The level of accounts receivable will most likely increase as (M) a. Has no need for a stated discount rate or credit period. Deterioration of aging and increase in the level of receivables. RPCPA 1095. in effect. financing more than just the purchaser’s inventory needs. RPCPA 1094 62 . Cash sales increase. CMA 0691 1-7 WORKING CAPITAL FINANCE *. Decrease in cash. RPCPA 0594 60 . Deterioration of aging of receivables only. because of the campaign to achieve volume targets. AICPA. Cash sales increase and number of says sales. Will have a lower level of accounts receivable than those companies whose credit period is shorter than the purchaser’s operating cycle. Increased payables and increased bank loan. Credit limits are expanded. c. what change would be expected on the balance sheet of its customer if the firm went to a net cash 30 policy? (M) a. The two credit proposals being considered are as follows: Proposal A Proposal B Increase in sales $500. the general manager has waived the credit block policy in a number of instances involving big volume accounts. provides for the imposition of credit block when the credit line is exceeded and/or the account is past due. c. During the month. CMA 1296 1-6 61 . Which one of the following statements is most correct if a seller extends credit to a purchaser for a period of time longer than the purchaser’s operating cycle? The seller (E) a. b. D. and a decrease in the investment in accounts receivable. Can be certain that the purchaser will be able to convert the inventory into cash before payment is due.MANAGEMENT ADVISORY SERVICES d. the company’s (E) A. An increase in the operating cycle. Increased receivables. e. Decrease in collections during the month the move was done.000 RPCPA. Working capital has increased. Based on this information. a reduction of the investment in accounts receivable. CMA & CIA EXAMINATION QUESTIONS Page 14 of 39 . C. Is. a decrease in the amount of bad debts. Credit limits are expanded. d. CMA 1285 1-6 *. The average collection period has decreased. 58 . cash sales increase. Based upon this information. d. b. Gross profit has declined. credit sales increase. D. an increase in discounts taken. b. An increase in sales resulting from an increased cash discount for prompt payment would be expected to cause (M) A. The bad debt loss percentage has increased. C. The credit and collection policy of Amargo Co. A decrease in purchase discounts taken. Matching the maturities of assets and liabilities reduces risk.000 $90. there is a decrease in the distribution level of your product. CMA 0697 1-14 *. A company serves as a distributor of products by ordering finished products once a quarter and using that inventory to accommodate the demand over the quarter. C.000 Desired return on sales 15% 15% Currently. All of the above b. Greater the risk that it will be unable to meet principal and interest payments. D. Trade credit c. The yield curve has traditionally been downward sloping. Pledging. Conversion of accounts receivable to bad debt on financial statements for accounts that are long overdue. can be raised from a number of different sources including a. Debt capital be it long-term or short-term in nature. 67 . CMA 0688 1-17 63 . If it plans to ease its credit policy for customers. Current bad debt experience. The account receivable level is improving. Reduced to offset the increased cost of carrying accounts receivable. Adjustment of inventories on financial statements for supplies that have become obsolete. D. c. Unaffected if the JIT inventory control system is used. A firm that borrows heavily long-term is more apt to be unable to repay the debt than a firm that borrows heavily short-term. B. Current ratio increases. C. CMA & CIA EXAMINATION QUESTIONS B&M . Impact on the current customer base of extending terms to only certain customers. An analysis to compare these two proposals for the change in credit policy would include all of the following factors except the a. B. CIA 0589 IV-53 SHORT-TERM FINANCING Short-term & Long-term Financing *. and financing requirements also remain constant. None of the above RPCPA 1091 66 . Hedging. Short-term borrowing using accounts receivable and characterized by the fact that the lender not only has a claim against the receivables but also has recourse against the borrower is A. AICPA. B. Your advise will be favorable if (E) a. CIA 0590 IV-50 D. Assumes full risk of default D. Factoring. c. Has a lien on the receivables and recourse to the borrower C. Selling. In a loan arranged through the assignment of accounts receivable the lender: A. Factoring is the (E) A. Accepts the actual receivable to be collected B. Likelihood of having idle liquid assets increases. b. D. C. Sales remain constant over the year. so the company can afford the carrying cost of receivables. the A. respectively. Unaffected if safety stock is part of the current quarterly order. the amount of products ordered for its inventory every quarter will be A.MANAGEMENT ADVISORY SERVICES WORKING CAPITAL FINANCE Contribution margin 20% 20% Bad debt percentage 5% 5% Increase in operating profits $75. The projected margin from increased sales will exceed the cost of carrying the incremental receivables. payments terms are net 30. As Finance Director you are asked to advise on the propriety of relaxing the credit standards in view of stiff competition in the market. Bank loan covenants on days’ sales outstanding. d. d. Bank loan d. as a company increases the amount of short-term financing relative to long-term financing. Which of the following statements is most correct? (M) Page 15 of 39 . Selling of accounts receivable by one company to another. c. Selling of inventory by one company to another. b. Firms generally choose to finance temporary assets with short-term debt because (M) a. Brigham e. RPCPA 0594 Factoring 64 . b. The proposed payment terms for Proposal A and Proposal B are net 45 and net 90. A strict credit and collection policy is in place in Star Co. C. Increased to accommodate higher sales levels. Short-term interest rates have traditionally been more stable than long-term interest rates. In general. B. All of the above RPCPA. Leverage of the firm increases. and a more aggressive stance in necessary to retain market share. CIA 0593 IV-53 Assignment 35. The competitor will do the same thing to prevent lost sales. Pledging 65 . Cost of funds for Ryan. d. B. The items purchased have a lower price. of not taking trade discounts is CIA 1195 IV-53 Discount % 360 A. The discount percentage is lower. but the use of short-term debt would probably increase the firm’s risk. 75 . D. The cost of not taking the discount is higher for terms of 2/10. With trade terms of 2/15. c. B. CIA 0597 IV-51 74 69 . a firm’s expected ROE would probably be higher if it financed with short-term rather than with long-term debt. Installment loans. then we would expect to see zero accounts payable on its balance sheet. A. Long-term debt. Commercial paper. c. The cost of not taking a cash discount is generally higher than the cost of a bank loan. Brigham e. c. d.Discount period 100  Discount % 360 x Discount % Days credit is outstanding . If a firm that can borrow from its bank buys on terms of 2/10. A short-term loan can usually be obtained more quickly than a long-term loan but the penalty for early repayment of a short-term loan is significantly higher than for a long-term loan. D. Brigham . D. Debentures. Mortgage bonds. B. the buyer receives 45 days of free credit. when (D) A. A company obtaining short-term financing with trade credit will pay a higher percentage financing cost. A small retail business would most likely finance its merchandise inventory with A.Discount period x 100 . A source of long-term financing to the seller. The cost of not taking a 2/10. A short-term loan can usually be obtained more quickly than a long-term loan. CMA & CIA EXAMINATION QUESTIONS C. Conservative firms generally use no short-term debt and thus have zero current liabilities. d. Accounts receivable. Which one of the following statements about trade credit is correct? a. Subject to risk of buyer default. Which one of the following statements concerning cash discount is correct? (E) A. D. Which of the following statements is most correct? (M) a. on an annual basis.Discount % Days credit is outstanding . if the discount is not taken. and riskiness of short-term versus long-term credit are dependent on the type of credit that is actually used. Which one of the following is a spontaneous source of financing? A. A chattel mortgage. CMA 1295 1-9 71 . Prepaid interest. C. d. B. e. and if it must pay by Day 30 or else be cut off. CMA 1296 1-12 72 . Under normal conditions. b. AICPA. The correct equation for calculating the approximate percentage cost. net 30 cash discount is usually less than the prime rate. Mortgage bonds.Discount period Discount % Days credit is outstanding .Discount % 360 73 . b. net 60 than for 2/10. D. Statements about the flexibility. C. although short-term debt has other advantages that make it desirable as a financing source. Usually an inexpensive source of external financing. x 100 . cost. B. Flexibility is an advantage of short-term credit but this is somewhat offset by the higher flotation costs associated with the need to repeatedly renew short-term credit. Not an important source of financing for small firms. If one of your firm’s customers is “stretching” its accounts payable. net 30. Under normal conditions the shape of the yield curve implies that the interest cost of short-term debt is greater than that of long-term debt. C. b. Trade credit. Which one of the following provides a spontaneous source of financing. Short-term debt is often less costly than long-term debt and the major reason for this is that short-term debt exposes the borrowing firm to much less risk than long-term debt. B. Which one of the following financial instruments generally provides the largest source of shortterm credit for small firms? RPCPA. Trade Credit 68 . C. Accounts payable. net 60. A line of credit.Discount period Discount % 360 x 100 Days credit is outstanding . . The items purchased have a higher price. (E) a. CMA 1295 1-7 C. d. Notes payable.MANAGEMENT ADVISORY SERVICES WORKING CAPITAL FINANCE a. Commercial paper. The supplier offers a longer discount period. Trade credit. net 30. D. b. 70 CMA 1295 1-15 CMA 1289 1-20 . A terminal warehouse receipt loan. this may be a nuisance but does not represent a real financial cost to your firm as long as the firm periodically pays off its entire balance. CMA 0689 1-17 Page 16 of 39 . everything else being equal. but the cost of short-term debt is likely to be higher than that of long-term debt. c. the cheapest source of short-term financing in this situation is (D) A. Often prearranged as a line of credit D. d. The prime lending rate of commercial banks is an announced rate and is often understated from the viewpoint of even the most credit-worthy firms.MANAGEMENT ADVISORY SERVICES WORKING CAPITAL FINANCE B. Usually repaid in increasing amounts over the term of the loan C. Often secured by a factoring agent B. 1/10. Often secured by commercial paper C. or a 14. Which of the following describes short-term bank loans? A. Which of the following statements is most correct? (M) a. Experience of personnel. B. None of the above B&M 76 . Competitive cost of services provided. The suppliers' terms are as follows: Fort Co. Almost never secured by inventory C. d. Brigham 22. Not significantly related to long-term rates. Which one of the following requirements always results in a higher effective interest rate? A. The bank. C. Riley Manufacturing Co. 3/15. c. Inc. Almost never secured by accounts receivable D. has a temporary need for funds. The prime rate is the (D) A. Cannot be from an international bank because of Federal Reserve System regulations E. b. Convenience of location. net 90 Using a 360-day year. Usually repaid in level amounts over the term of the loan C. b. Rate charged on business loans to borrowers with high credit ratings. this implies the company has 10 days of free trade credit. Size of the bank’s deposits. CMA 0691 1-5 79 . 2/15. Short-term interest rates are a. If unsecured. None of the above B&M 23. e. CMA 1283 1-25 Bank Loans . banks often require borrower to "clean up" the loan for 1 month in the year RPCPA. Inc. Accruals are an expensive way to finance working capital. A conservative financing policy is one in which the firm finances all of its fixed assets with long-term capital and part of its permanent current assets with short-term. Borrower is often obliged to maintain compensating balance B&M Interest rates 78 . d. c. Cannot be from an international bank because Federal Reserve System regulations prohibit Eurodollar borrowing E. D. Merkle. net 60 Shad. c.75% per annum renewable discount loan from its bank for 3 months. Which of the following statements describes bank term loans? A. Typical maturity is between 2 and 4 years B. Brigham 77 . Loyalty and willingness to assume lending risks. Lower than long-term rates during periods of high inflation only. Effective cost of a commercial bank loan. Which of the following describes short-term bank loans? A. Size of the commitment fee on a commercial bank loan. Statements a and b are correct. nonspontaneous credit. Inc. Page 17 of 39 . Which one of the following aspects of banks is considered most relevant to businesses when choosing a bank? (M) a. Borrower is often obliged to have the bank mange its corporate cash accounts B&M 25. Usually higher than long-term rates. Interest rate is usually fixed D. AICPA. CMA & CIA EXAMINATION QUESTIONS Term Loan 24. B. Shad. Usually lower than long-term rates. B. If a company receives trade credit under the terms 2/10 net 30. A floating rate for the loan period. b. A covenant that restricts the issuance of any new unsecured bonds during the existence of the loan. C. net 30 Riley Manufacturing Co. Effective cost of commercial paper. Management is trying to decide between not taking discounts from one of their three biggest suppliers. Interest rate is usually fixed D. CMA 0688 1-18 80 . D. Fort Co. Typical maturity is between 1 and 2 years B. Which of the following statements describe bank term loans? A. B. Compensating balances are essentially costless to most firms. All of the above B. not to individuals. A and B C.000 List B Simple Simple Discount Discount Compensating Balance 82 . A compensating balance A. D.000 $20. CMA 1280 1-3 Nominal Interest Rate 81 . C. Interest Average borrowed amount D. Borrowed amount Borrowed amount . CMA & CIA EXAMINATION QUESTIONS Page 18 of 39 . The absence of a charge for any unused portion in the line of credit. The imposition of a compensating balance with an absolute minimum that cannot be met by current transaction balances. b. Firms provide the following as security for short-term loans: A. The loans are transferred to new lenders RPCPA. (E) CIA 0595 IV-50 A. Compensating balance. List A $18. Loan syndications B&M 33. RPCPA 0596 34. then the effective cost of any loan requiring such a balance is increased. Precautionary balance. . d. Compensating balance requirements apply only to businesses. D. Banks are prohibited from earning interest on the funds they force businesses to keep as compensating balances. Loan sales by commercial banks may take the form of: A. Transactions balance. WIP inventory is relatively easy to sell because it does not represent a raw material or a finished product. In assessing the loan value of inventory. Inventories D. c. Securities B. When banks have to make large loans. Golden umbrella B. Discounted interest is based on the borrowed amount but is paid in advance. None of the answers are correct. Loan participations are different from syndicated loans in that: A. WIP represents a lower investment by a corporation as opposed to other types of inventories.000 loan with a 10% nominal interest rate provides the borrower with the use of <List A> if interest is charged on a <List B> basis. c. The lead bank provides a "certificate of participation" to each bank B. B. C. C. d. WIP inventory usually has the highest loan value of the different inventory types. A one year. Collateral *. Speculative balance. because those firms would normally have such funds on hand to meet transactions needs anyway.MANAGEMENT ADVISORY SERVICES C. CIA 1190 IV-49 83 . Is an amount paid by financial institutions to compensate large depositors. a banker will normally be concerned about the portion of inventory that is work-in-process because (E) a. If the required compensating balance is larger than the transactions balance the firm would ordinarily hold. Loan participations E. Compensates a financial institution for services rendered by providing it with deposits of funds. b. All of the above B&M Syndicated Loan 30. The group is called a: A. D. Which of the following statements is most correct? (M) a. AICPA. CMA 0688 1-13 B. The formula for calculating the discounted interest rate for a 1-year loan is (E) Gleim Interest Interest A. Accounts receivables C. Bank holding company C. Each participating bank has a separate loan agreement with the borrower C. C. D.Interest B. WIP generally has the lowest marketability of the various types of inventories. $20. Loan assignments D. they form a group of banks for the purpose of making the loan. A minimum checking account balance that a firm must maintain with a commercial bank is a A. Syndicate D. Conglomerate B&M Loan Sales by Commercial Banks 32.000 $20. Is used to compensate for possible losses on a marketable securities portfolio.000 $22. Is a level of inventory held to compensate for variations in usage rate and lead time. Brigham WORKING CAPITAL FINANCE Effective Rate 84 . chattel mortgage. Is readily available to almost all companies. chattel mortgage. CMA 0696 1-14 37. b. Agency securities. RPCPA. bankers' acceptances. This market provides a broad distribution for borrowing. bankers' acceptances. b. Commercial paper . C. C. B. Commercial paper can be issued by virtually any firm so long as it is willing to pay the going interest rate. Which of the following statements is incorrect? (E) a. after which both the banker and the seller record the transaction on their respective books. None of the above WORKING CAPITAL FINANCE B&M Banker’s Acceptance 85 . revolving credit. B. B. Bonds C. and line of credit. Debentures D. The borrower avoids the expense of maintaining a compensating balance with a commercial bank. C. Short-term.e. Brigham e. Added risk can stem from greater variability of interest costs on short-term debt. Has an interest rate lower than Treasury bills. and line of credit. B. D. The credit instrument known as a banker's acceptance A. Is a time draft payable on a specified date and guaranteed by the bank. Bankers' acceptances. Has a maturity date greater than 1 year. AICPA. Offers security. Repurchase agreements. CIA 0596 IV-41 Unsecured Credit 86 . Which one of the following responses is not an advantage to a corporation that uses the commercial paper market for short-term financing? a. bankers' acceptances. Ordinarily does not have an active secondary market. and commercial paper. Revolving credit. Commercial paper is sold to money market mutual funds. d. i. d. as well as to other financial institutions and nonfinancial corporations. Is usually cheaper than a commercial bank loan. A conservative approach to working capital will result in all permanent assets being financed using long-term securities. C. This market provides more funds at lower rates than other methods provides. CMA 1289 1-22 90 . D. 88 . b.. line of credit. Commercial paper A. The following forms of short-term borrowing are available to a firm:  Floating lien  Factoring  Revolving credit  Chattel mortgages  Bankers' acceptances  Lines of credit  Commercial paper The forms of short-term borrowing that are unsecured credit are A. D. B&M Which of the following statements concerning commercial paper is incorrect? (E) a. Accruals represent a source of “free” financing in the sense that no explicit interest is paid on these funds. chattel mortgage. D. CMA 0689 1-13 B. 91 . strong firms. Is a method of sales financing in which the bank retains title to the goods until the buyer has completed payment. unsecured promissory notes issued by large firms are known as A. The risk to the firm of borrowing with short-term credit is usually greater than with longterm debt. and commercial paper. There are no restrictions as to the type of corporation that can enter into this market. Page 19 of 39 . CMA 1286 1-35 D. Calls for immediate payment upon delivery of the shipping documents to the bank's customer and acceptance of goods by the bank. Is usually sold only through investment banking dealers. collateral. Commercial paper generally carries an interest rate below the prime rate. Involves an invoice being signed by the banker upon receipt of goods. d. The principal advantage of using commercial paper as a short-term financing instrument is that it (E) A. Factoring. Can be purchased without commission costs. CMA & CIA EXAMINATION QUESTIONS CMA 0691 1-10 89 . Floating lien. Commercial Paper 87 . c. Commercial paper can be issued by virtually any firm so long as it is willing to pay the going interest rate. to the lender. Large companies often raise short-term debt by selling: A. Floating lien. Commercial paper. Commercial paper is generally written for terms less than 270 days. Medium term notes B. Commercial paper is a type of unsecured promissory note issued by large. C.MANAGEMENT ADVISORY SERVICES D. c. c. Its effective rate is 20%. Blanket inventory lien. Accepting the 1-year loan at 15% with the compensating balance provisions. A warehouse receipt. and line of credit B. Aa D. A 20% compensating balance and regular interest. Baa D. 20% compensating balance required. Its effective rate is 17%. Which of the following is a commercial paper rating by Moody's? A. A bank has offered the company three alternatives: 1. or 3. A 1-year loan at 15% with the provision that it maintains 20% of whatever amount it borrows as noninterest-bearing compensating balances over the life of the loan. Trade credit is often the largest source of short-term credit. and trust receipt . The three basic forms of inventory loans include: A. The note. Accepting the 1-year loan at 18% with no other provisions. D. A manufacturing firm wants to obtain a short-term loan and has approached several lending institutions. A company is arranging debt financing for the purchase of a new piece of equipment that has a 5-year expected useful life. A 10% compensating balance and regular interest. A 1-year loan at 18% with no other fees. CIA 1196 IV-52 . Commercial paper.MANAGEMENT ADVISORY SERVICES e. Which of the following combinations of loan terms will be most attractive for the borrowing firm? A.The company would achieve the lowest cost of financing by (M) A. CIA 1188 IV-54 RPCPA. A company will receive cash from sales in 1 year that can be used to pay for materials. B. net 30 or a 30-day note with a 20% annual simple interest rate. P-1 B. Line of credit.000 at 17% with the provision that the bank will collect a 1% fee on the average amount of unused funds. An example of secured short-term financing is A. Short-term securities issued by the Federal Housing Administration are known as A. D. no compensating balance. A 10-year term loan with interest compounded monthly. None of the above B&M 96 B&M Agency Securities 92 . A short-term bank loan will have a higher effective financing cost if it has which combination of characteristics? A. CMA 0689 1-14 Secured Short-term Financing 93 . All of the above 13. A 20% compensating balance and discount interest. The note. CMA & CIA EXAMINATION QUESTIONS . Bankers' acceptances. A 5-year term loan with interest compounded quarterly. C. line of credit. The company expects to borrow no other funds. A revolving credit agreement. and trust receipt C. 2. CIA 0594 IV-51 97 .5%? A. field warehouse financing. C. Field warehouse financing. Blanket inventory lien.000 purchase price. It can either use its trade credit on $100. but it must borrow the full amount. it will receive a 20% discount off the $100. B. and what is its effective rate of interest (rounded to a whole percentage and using a 360-day year)? (M) A. Which of the following alternative financing arrangements has the lowest effective annual percentage rate if each has a quoted nominal rate of 9.000 of accounts payable with terms of 1/10. 20% compensating balance required. Discount interest. C. Simple interest. C. Page 20 of 39 . A 5-year term loan with interest compounded annually. B. Agency securities. Blanket inventory lien. line of credit. If the company pays the supplier immediately. and trust receipt D. C. C. Aaa B. Aaa C. Commercial paper. All of the potential lenders are offering the same nominal interest rate. D. B. D. A 10% compensating balance and discount interest. Discount interest. CIA 1191 IV-56 Inventory Loans 36. P-2 C. Its effective rate is 10%. CIA 1196 IV-45 98 B&M Alternative Financing Arrangements 94 . Which of the following is a commercial paper rating by Moody's? A. Simple interest. D. B. Repurchase agreements. Allowing the supplier to finance the materials and making payment at the end of 1 year. field warehouse financing. B. C. Which is the best alternative. but the terms of the loans vary. D. AICPA. The trade credit. B. A 10-year term loan with interest compounded semiannually. The trade credit. A guaranteed line of credit of $100. Its effective rate is 20%. WORKING CAPITAL FINANCE Brigham 95 Commercial Paper Rating 12. The supplier will allow payment in 1 year. no compensating balance. A corporation is currently experiencing cash-flow problems and has determined that it is in need of short-term credit. CMA & CIA EXAMINATION QUESTIONS Page 21 of 39 . Accepting the guaranteed line of credit at 17% with the fee required on the average amount of unused funds. CIA 1186 IV-43 RPCPA.MANAGEMENT ADVISORY SERVICES WORKING CAPITAL FINANCE D. AICPA. AICPA. CMA & CIA EXAMINATION QUESTIONS Page 22 of 39 .MANAGEMENT ADVISORY SERVICES WORKING CAPITAL FINANCE ANSWER EXPALANATIONS RPCPA. Answer (A) is incorrect because the growing firm is more apt to emphasize production rather than protecting against technical insolvency by maintaining a high level of working capital. Answer (C) is correct. This policy results in a lower current ratio. DISCUSSION: (C) Fluctuating curent assets can often be financed with short-term debt because the periodic liquidition of the assets provides funds to pay off the debt. This policy maximizes return on investment at the price of the risk of minimal liquidity. Answer (D) is incorrect because the company needs its profits to invest in new production equipment in order to grow. financing permanent current assets with shortterm debt is a risky strategy because the assets may not be liquidated in time to pay off the debt at maturity.current liabilities). an increasing ratio of current to noncurrent higher returns on long-term assets in order to guard against short-term cash flow problems. Unfortunately. management keeps the investment in working capital at a minimum. minimizes the risk that the company will not be able to meet its obligations as they fall due. Answer (C) is incorrect because Clay is subject to greater liquidity risk than Lott since it has greater short-term debt. Answer (A) is incorrect because a decrease in the acid-test ratio suggests an aggressive policy. Answer (B) is correct. A conservative company wants a higher acid-test ratio. 2 . REQUIRED: The true statement about a more conservative working capital policy. Thus. Answer (D) is incorrect because financing fluctuating current assets with short-term debt is not as risky as financing permanent current assets with short-term debt.1 . Thus. and (B) are incorrect because it is not particularly risky to finance working captial needs from long-term debt sources. Answer (A) is incorrect because an increase in current liabilities relative to noncurrent liabilities would increase liquidity risk. Answer (C) is incorrect because the company will prefer to expend funds on capital goods. Failure to do so is a technical insolvency and can result in involuntary bankruptcy. Answer (D) is incorrect because a more conservative company would tend to finance by means of equity rather than debt capital. a growing company would want to invest its funds in capital goods and not in idle assets. The result is that the company forgoes the potentially higher returns available from using the additional working capital to acquire long-term assets. A conservative working capital policy is characterized by a higher current ratio (current assets/current liabilities) and acid-test ratio (quick assets/current liabilities). A conservative policy finances assets using long-term or permanent funds rather than short-term sources. holding current assets for purposes of paying bills is not profitable for a company because they usually offer a low return compared with longer-term investments. the skillful management of working capital requires a balancing of a firm’s . A conservative working capital policy minimizes liquidity risk by increasing working capital (current assets . increasing the current ratio. Answer (C) is incorrect because a decrease in the quick ratio signifies that quick assets ( cash. It is typified by a reduction in liquidity risk. whether by decreasing current liabilities or increasing current assets. Thus. that is. Answer (D) is incorrect because a conservative company seeks more liquid (marketable) investments. Answer (B) is incorrect because a conservative company wants working capital to be financed from long-term sources. However. When a firm has an aggressive working capital policy. DISCUSSION: (D) A conservative working capital policy results in an increase in working capital (current assets – current liabilities). Answer (B) is correct. 6 . REQUIRED: The true statement about the determination of the appropriate level of working capital. the company will increase current assets or decrease current liabilities. it is at greater risk of being unable to meet its maturing obligations. Answer (A) is incorrect because Clay's aggressive policy would result in more short-term debt. more liquid assets relative to liabilities. A conservative working capital management financing policy uses permanent capital to finance permanent asset requirements and also some or all of the firm's seasonal demands. receivables. REQUIRED: The working capital financing policy that subects a firm to the greatest risk of being unable to meet maturing obligations. DISCUSSION: (C) A company must maintain a level of working capital sufficient to pay bills as they come due. and marketable securities) are decreasing relative to current liabilities. 3 . 5 . current assets can be reduced. An aggressive policy entails financing some fixed assets and all the current assets with short-term capital. Lott's current ratio (current assets/current liabilities) will be high since its current liabilities will be relatively low. Thus. If assets can be converted to cash more quickly. Lott's conservative policy would produce more long-term debt or equity financing. 4 . Hence. the more conservative company would have less interest expense. with attendant renewal problems and high risk. Thus. Thus. Answer (B) is incorrect because a decrease in the normal operating cycle permits a lower level of working capital. Answers (A). Answer (C) is incorrect because total assets and total liabilities are not components of working capital. Answer (C) is incorrect because the purchase of temporary investments does not affect total current assets. Answer (D) is correct. Working capital equals current assets minus current liabilities. Answer (A) is incorrect because a prepayment of expenses does not change current assets or current liabilities. Although depreciation is a noncash expense. only current items are included. and the result is an increase in working capital. Answer (B) is incorrect because buying a new plant with a 20-year mortgage has no effect on current assets or current liabilities. 9 . which are a cash expense. A lockbox speeds collections of receivables. (B) and (C) are incorrect because both working capital and the current ratio increase. The borrowing of cash by incurring short-term debt increases current assets by the same amount as it increases current liabilities. not in working capital financial management. Answer (D) is incorrect because the collection of a receivable has no effect on total current assets. Answers (A). Working capital is the excess of current assets over current liabilities. The receivable is replaced by an equal amount of cash. statement d is the appropriate choice. hence. The current ratio equals current assets divided by current liabilities. 12 . fixed assets are not a component. Refinancing a short-term debt with a long-term debt decreases current liabilities. Answer (E) is incorrect because maintaining a high level of liquid assets is usually not profitable. Answer (C) is incorrect because short-term debt is usually less expensive than long-term debt. 8 . The purchase of a delivery van for cash reduces current assets and has no effect on current liabilities. Answer (A) is incorrect because management of fixed assets is not a factor in working capital management. another current asset. 13 . Answer (A) is incorrect because the purchases of the van and treasury stock affect working capital. it does affect taxes. 10 . Answer (C) is incorrect because the purchases of the van and treasury stock but not the issuance of shortterm debt affect working capital. Answer (B) is incorrect because working capital refers to the difference between current assets and current liabilities. only current items are included in the concept of working capital. Working capital is computed by deducting total current liabilities from total current assets. Answer (B) is correct. REQUIRED: The effect on working capital and the current ratio of issuing common stock for cash. Answer (B) is incorrect because capital structure and dividend policy are factors involved in capital structure finance. 11 . Selling stock for cash increases current assets and stockholders’ equity. the purchases of the van and treasury stock affect working capital. DISCUSSION: (A) Net working capital is defined as the difference between current assets and current liabilities. cash is replaced by temporary investments. 7 . Working capital is a measure of short-term solvency. another current asset. with no effect on current liabilities. and the result is an increase in working capital. Answer (B) is correct. Refinancing a short-term note with a two-year note payable decreases current liabilities. Answer (D) is incorrect because the purchase of financial assets held for trading does not affect total current assets. it doesn’t ensure that petty cash will be safe. Thus.desire for profit with its need for adequate liquidity. Statements a and c are correct. Answer (D) is correct. thus increasing working capital. Answer (A) is incorrect because a prepayment of expenses does not change current assets or current liabilities. Answer (A) is incorrect because a cash payment of payroll taxes decreases current assets and current liabilities by equal amounts. therefore. . DISCUSSION: (D) Working capital equals current assets minus current liabilities. Answer (C) is incorrect because the acquisition of land for common shares does not affect either current assets or current liabilities. cash is replaced by the financial assets held for trading. Cash decreases by the same amount that prepaid rent increases. Answer (B) is incorrect because the purchases of the van and treasury stock but not the issuance of short-term debt affect working capital. REQUIRED: The definition of net working capital. The purchase of treasury stock decreases current assets but has no effect on current liabilities. Cash decreases by the same amount that prepaid rent increases. Answer (C) is incorrect because cash collection of an account receivable increases one current asset and decrease another by the same amount. it will have no effect on working capital. Refinancing a short-term debt with a long-term debt decreases current liabilities. The result is an increase in working capital and the current ratio. Answer (D) is incorrect because shareholders’ equity is not a component of working capital. Working capital is the excess of current assets over current liabilities. Answer (D) is incorrect because common stocks do not have sufficient liquidity to serve as short-term cash resources. Answer (A) is incorrect because a transactions balance is associated with routine payments and collections. to guard against unforeseen fluctuations in cash flows. Answer (A) is incorrect because the three major motives for holding cash. are for transactional. despite high risk and low liquidity. Answer (C) is incorrect because investments in Treasury bonds do not have sufficient liquidity to serve as short-term assets. founder of Keynesian economics. Answer (B) is incorrect because a compensating balance is the minimum amount on deposit at a bank to compensate the bank for providing loans and other services. concluded that there were three major motives for holding cash: for transactional purposes as a medium of exchange. Cash and short-term investments are held because of their ability to facilitate routine operations of the company. DISCUSSION: (D) Financing permanent inventory build-up which is essentially a long-term investment. 15 . Answer (A) is incorrect because most companies are not in business to earn high returns on liquid assets (i. and to take advantage of opportunities for bargain purchases. Answer (C) is incorrect because matching of asset and liability maturities is a moderate policy that minimizes risk. and speculative purposes. they are held to facilitate operations). A company will hold cash and marketable securities to facilitate business transactions because cash is a primary medium of exchange.14 . However. Answer (B) is incorrect because the holding of cash and cash-like assets is not a major factor in controlling taxes. to satisfy compensating balance requirements imposed by lenders. precautionary purposes. A good cash management system maximizes disbursement float and minimizes collections float. REQUIRED: The false statement about working capital. and to provide a precautionary balance for security purposes.. trade credit is a major source of funds for small firms. firms are more likely to hold trading securities or rely on easy access to credit to take advantage of bargains or to guard against unforeseen fluctuations in cash flows. Sufficient liquidity must be available to meet payments as they come due. Answer (A) is incorrect because current liabilities. according to Keynesian economics. precautionary. . 16 . 19 . and speculative purposes. An aggressive policy uses short-term relatively lowcost debt to finance the inventory buildup. DISCUSSION: (E) Cash and short-term investments are crucial for a firm’s continuing success. Financing inventory with long-term debt increases the current ratio and accepts higher borrowing costs in exchange for greater liquidity and lower risk. The other statements are false. Answer (D) is correct. and speculative purposes (but only during deflationary periods). Cash and near-cash items are also held to meet future needs. precautionary. to compensate banks for providing loans and services. 18 . e. are for transactional. Answer (B) is incorrect because the three major motives for holding cash. Answer (D) is correct. Therefore.. precautionary. Increases in interest rates raise the opportunity cost of idle cash. It focuses on high profitability potential. liquidity and safety are the primary concerns of the treasurer when dealing with a highly liquid assets. Answer (C) is incorrect because a precautionary balance consists of reserves for unforeseen fluctuations in cash flows. A very efficient cash management system could allow a firm to operate with positive net float where the firm has a negative checkbook balance at most times but still does not bounce its checks. Answer (C) is incorrect because cash is held to meet future needs. Answer (A) is incorrect because cash is held to satisfy compensating balance requirements. Cash is usually not held in an attempt to earn maximum returns on investment because cash and marketable securities are not usually the highest-paying investments. Answer (B) is incorrect because liquid investments tend to have lower returns. with longterm debt is a moderate or conservative working capital policy. A well-designed lockbox system minimizes collections float which would increase a firm’s net float. according to Keynesian economics. These assets are not held for purposes of achieving investment returns. Answer (C) is incorrect because the three major motives for holding cash. REQUIRED: The primary concern when managing cash and short-term investments.g. Answer (D) is correct. are for transactional. according to Keynesian economics. liquid assets are subject to significant control risk. and speculative purposes.e. An aggressive policy reduces the current ratio and accepts a higher risk of short-term lack of liquidity. The expectation is that cash flows from the assets will be available to meet obligations for the liabilities. John Maynard Keynes. 17 . Answer (B) is incorrect because cash is held to facilitate transactions. At the same time. A firm prefers to write checks. maximizing its disbursement float and increasing its net float. Companies hold cash to facilitate routine transactions. A check is the most common form of draft. therefore the larger the disbursements float and the lower the collections float the better the cash management system. A compensating balance is a minimum average or absolute amount that must be maintained in a bank account. Answer (A) is incorrect because a lock-box system is not related to compensating balances. A bank checks these mailboxes several times a day. Payment by draft. the outflow is delayed until the check clears the drawee bank. a lockbox system. 22 .20 . and the use of a local post office box are techniques used to accelerate cash receipts. Answer (B) is incorrect because a lock-box system is a process by which payments are sent to a bank's mailbox. a draft can be used to delay the outflow of cash. a three-party instrument in which the drawer orders the drawee to pay money to the payee. which is checked during normal post office hours. The use of an ACH facilitates concentration banking. DISCUSSION: A draft is a three-party instrument in which one person (the drawer) orders a second person (the drawee) to pay money to a third person (the payee). a lockbox system. to which customers send payments. and the use of a local post office box are techniques used to accelerate cash receipts. 21 . Answer (A) is incorrect because concentration banking. not outflows. REQUIRED: The working capital technique that delays the outflow of cash. EDI expedites cash payments. 24 . These local banks operate the company’s lockboxes and thus serve as collection points. Net float = Disbursements float . often in numerous locations around the country. DISCUSSION: (A) An ACH electronic funds transfer (EFT) is an electronic payment to a company’s account of a concentration bank. Answer (D) is incorrect because it is a common . Answer (D) is incorrect because concentration banking. A draft can be dated on the due date of an invoice and will not be processed by the drawee until that date. A lockbox is used to speed cash collections. Check float arises from the delay between an expenditure and the clearing of the check through the banking system. and lockbox systems. A check is the most common type of draft. Thus. Answer (D) is correct. Thus. Answer (C) is incorrect because the use of a lock-box system entails sending checks through the mail to a post office box. The payee receives the money almost instantaneously. Answer (B) is incorrect because a check is not involved in an EFT. Answer (C) is correct. and funds received are immediately deposited to the company's account without first being processed by the company's accounting system. Consequently. REQUIRED: The definition of an automated clearing house (ACH) electronic transfer. It is an instrument payable on demand in which the drawee is a bank. Answer (D) is incorrect because an electronic funds transfer results in an immediate deduction from the payor’s bank account. Thus. Answer (B) is incorrect because a lockbox system is a means of accelerating cash inflows. Answer (D) is incorrect because an EFT is not a check-like instrument. is a means of slowing cash outflows. The transfer of funds to the concentration bank allows the company to take advantage of economies of scale in cash management. thereby eliminating the necessity of writing a check earlier than the due date or using an EFT. A company maintains mail boxes. A concentration bank is a large bank to which a company transfers funds from local depository banks. it does not reduce the risk of losing checks in the mail. Various methods of accelerating cash collections include decentralized collection outposts (normally one in each Federal Reserve District). electronic funds transfers. ACHs are electronic networks operated by the Federal Reserve (except for the New York regional ACH association) that guarantee 1-day clearing. Answer (C) is incorrect because an ACH transfer involves the actual transfer of funds electronically.Collections float. it is not just a computer generated document. Answer (A) is incorrect because it is a common method of accelerating cash collections. A lock-box system accelerates the inflow of funds. 23 . If a firm’s outflows come due early in the month rather than uniformly this will necessitate a large line of credit. Answer (A) is incorrect because factoring is the sale of receivables and therefore concerns cash inflows. 25 . Hence. it is not a means of accelerating cash collections. Answer (B) is correct. Answer (C) is incorrect because EDI is the communication of electronic documents directly from a computer in one entity to a computer in another entity. This requirement means that less cash is available to the depositor. a compensating balance may be required by a covenant in a loan agreement that requires a company to maintain a specified balance during the term of the loan. thereby eliminating float. centralized banking for all company branches to avoid having to maintain minimum balances in several locations. thereby hastening availability of the funds. Answer (B) is incorrect because it is a common method of accelerating cash collections. Answer (D) is incorrect because the costs of issuing securities relate to capital structure finance. Answer (C) is incorrect because investments in Treasury bonds do not have sufficient liquidity to serve as short-term assets. Answer (C) is incorrect because the cash inflows have not been received. If the company reduces its DSO. 29 . Thus. 30 . in the case of production. Answer (C) is incorrect because the relationship between current assets and current liabilities concerns many factors other than cash management. Thus. the cash conversion cycle. Answer (A) is incorrect because most companies are not in business to earn high returns on liquid assets (i. 32 . Answer (D) is incorrect because the cash inflows have not been received. a cash management model determine how much of a firm’s liquidity should be held as cash and how much in the form of marketable securities. Statement b is true. and sales do not. in principle they may be treated similarly. begins with the cash purchase of raw materials and ends with the collection of accounts receivable. The length of the cycle is from the cash outflow to the cash inflow. Answer (C) is incorrect because special handling of large checks is a cost-effective way to deposit large amounts. These assets are not held for purposes of achieving investment returns. If the company pays its bills sooner. If inventory increases. Therefore. Answer (D) is correct. a greater liquidity means less risk of being unable to meet obligations when they are due. Sufficient liquidity must be available to meet payments as they come due. Answer (D) is correct. Answer (A) is incorrect because multiple collection centers throughout the country will reduce the time required to receive cash in the mail. therefore. 33 .method of accelerating cash collections.Statement a is false. The trade-off between risk and return must be considered because liquid assets are usually less profitable than less-liquid alternatives. they are held to facilitate operations). The optimal amount of cash to be raised by selling securities is calculated by a formula similar to that used to determine the economic order quantity for inventory.. so (D) is a better answer. DISCUSSION: (D) Because cash and inventory are both non-earning assets. which increase its cash conversion cycle. For example. Compensating balances are either (1) an absolute minimum balance or (2) a minimum average balance that bank customers must keep at the bank. liquid assets are subject to significant control risk. Answer (B) is incorrect because the holding of cash and cash-like assets is not a major factor in controlling taxes. These are generally required by the bank to compensate for the cost of services rendered. Answer (C) is correct. REQUIRED: The true statement about cash management models. Answer (A) is incorrect because financial leverage concerns the extent to which debt financing is used. however. At the same time. is to hold marketable securities that do earn interest or dividends. The alternative to holding cash. Answer (B) is incorrect because operating leverage concerns the proportion of fixed operating costs. liquidity and safety are the primary concerns of the treasurer when dealing with highly liquid assets. Delaying payments to suppliers increases the length of the cash conversion cycle. Cash conversion is the process of converting cash to products and back to cash. 31 . . 28 . Cash and short-term investments are held because of their ability to facilitate routine operations of the company. Maintaining compensating balances will not accelerate a company's cash inflows because less cash will be available even though the amount of cash coming in remains unchanged. 26 .Statements a and b are correct.e. Answer (B) is incorrect because the level of marketable securities is in part determined by cash needs. Answer (B) is incorrect because direct deposit by customers into a lock-box also speeds cash into company accounts. more cash is being “tied up” in inventory so the cash conversion cycle is increased. so it reduces the cash conversion cycle. California customers of a New York firm would make payment to a West Coast center. Answer (A) is correct. Answer (B) is incorrect because the cash inflows have not been received. it is collecting its accounts receivables more efficiently. Answer (A) is correct. However. Thus. Answer (A) is incorrect because credit and collection policies concern receivables and are not influenced by an EOQ model for inventory management. not reduced. Statement c is false. 27 . Cash and short-term investments are crucial to a firm's continuing success. the company would receive the cash two or three days sooner. it uses its cash to pay off accounts payable. statement d is the appropriate choice. regardless of the amount withdrawn. Answer (C) is incorrect because a high (low) opportunity cost results in a lower (higher) optimal cash balance. whereas high (low) transaction costs result in a higher (lower) optimal cash balance. such as the Federal Housing Administration. whereas high (low) transaction costs result in a higher (lower) optimal cash balance. Answer (D) is incorrect because these are factors in the determination of cash flow. Answer (D) is incorrect because Aaa corporate bonds lack the liquidity necessary to be a cash substitute. Answer (A) is incorrect because securities issued by a federal agency are first backed by that agency and secondarily by the U. Answer (D) is incorrect because commercial paper is unsecured. Common stock can also be a risky investment.S. A Treasury bill is a short-term U. Relative yields are the rates of return of each security in comparison with other potential investments. Answer (C) is incorrect because the EOQ model assumes that the total demand for cash is known with certainty. Gold can also be a risky investment. the shorter-term the instrument. Answer (D) is incorrect because a high (low) opportunity cost results in a lower (higher) optimal cash balance. government. Taxability refers to the tax treatment of any income or loss from a security.C+ = 2(F)(T) k If: C+ = Cash to be raised T = Total cash needed for the period F = Cost of making a securities trade k = Opportunity cost of holding cash The optimal amount of cash to be raised by selling securities is inversely related to the rate of return forgone (opportunity cost) and directly related to the cost of the transaction. government obligation that is sold at a discount from its face value. the cash flow requirements cannot be random. . The operating and payment cycles are components of the cash conversion cycle. the amount of cash needed. Answer (B) is incorrect because common stock lacks the liquidity necessary to be a cash substitute. This cycle lasts from the time cash is paid for resources to the time cash is received in payment for goods sold. Answer (B) is correct. They are backed by a secondary promise from the government. A Treasury bill is highly liquid and nearly risk-free. The receivables conversion cycle lasts from the date of sale of finished goods to the date of cash collection. Thus. S. The marketable securities with the lowest default risk are those issued by the federal government because they are backed by the full faith and credit of the U. Only Treasury issues are insured by the U. Answer (A) is correct.S. whereas high (low) transaction costs result in a higher (lower) optimal cash balance. or in this case. Answer (C) is incorrect because gold lacks the liquidity necessary to be a cash substitute. and taxation is a pervasive issue that affects yield. 36 . the safer the instrument.g. Answer (C) is incorrect because these cycles are important in working capital management. and it is often held as a substitute for cash. 34 . Default risk is the risk that a borrower will be unable to make interest payments or principal repayments on debt (e. Answer (A) is correct. The inventory conversion cycle is the time between acquisition of resources and sale of finished goods. DISCUSSION: (D) The EOQ formula is a deterministic model that requires a known demand for inventory. the cash conversion cycle does not begin until the end of the payable deferral period. Answer (C) is incorrect because repurchase agreements could become worthless if the organization agreeing to make the repurchase goes bankrupt. 37 . REQUIRED: The assumption not made in the EOQ model. Answer (B) is incorrect because a high (low) opportunity cost results in a lower (higher) optimal cash balance. A payable deferral period is the lapse of time between purchase of resources and the date they are paid for.S. government. Risk and yield are fundamental concerns. Answer (A) is incorrect because the EOQ model assumes that the cost of a transaction is independent of the dollar amount of the transaction and interest rates are constant over the short-run. Aaa corporate bonds can also be risky investments. The model also assumes a given carrying (interest) cost and a flat transaction cost for converting marketable securities to cash. not securities selection criteria. Agency securities are issued by agencies and corporations created by the federal government. Thus. bonds purchased from a financially troubled company have high default risk). beginning with the first dollar. Answer (B) is incorrect because these matters are a financial manager's overall responsibility. Answer (B) is incorrect because the EOQ model assumes an opportunity cost is associated with holding cash.. 38 . 35 . on which the company doing business with the bank's customer draws a time draft. by definition. Answer (C) is correct. The local bank forwards the time draft to the bank customer for payment. However. insurance. A certificate of deposit (CD) is a form of savings deposit that cannot be withdrawn before maturity without incurring a high penalty.S. Answer (D) is incorrect because U. their yield is less than that of commercial paper and bankers' acceptances because they are less risky. Commercial paper is debt and thus has priority over stockholders' claims.S. The maximum maturity allowed without SEC registration is 270 days.000. negotiable certificates of deposit.S. It is usually initiated by a bank's irrevocable letter of credit on behalf of the bank's customer.S. Answer (A) is incorrect because these deposits are outside the control of U. Answer (D) is incorrect because an increase in interest rates could cause a substantial loss in principal. 44 . bankers' acceptances. 42 . which are U. always sold in some country other than the one in whose currency the bond issue is denominated.S. Answer (B) is incorrect because these deposits are outside the control of U. This amount is then available for lending by the foreign bank to its customers.S. A banker's acceptance is a unique credit instrument used to finance both domestic and international "self-liquidating" transactions. the depositors still hold claims denominated in dollars. Capital markets trade shares and long-term debt. Answer (B) is incorrect because negotiable CDs are regulated. Answer (C) is incorrect because negotiable CDs are typically issued in a denomination of $100.. 43 . authorities because they are in banks outside the U. Treasury bonds are long-term capital market securities. Answer (A) is incorrect because municipal bonds are rarely considered marketable securities in that they constitute long-term debt.39 . rates on equivalent instruments.S. Funds are borrowed or lent for short periods (less than one year) in money markets. authorities because . Because of diversification. However. do not apply. Eurobonds are. Answer (D) is correct.S. Of the choices given. Answer (B) is incorrect because the transactions cost of bankers' acceptances is high. than domestic U. Answer (C) is correct. Answer (A) is incorrect because mortgages are long-term capital market securities. Answer (D) is incorrect because common stock does not have as high a priority in company assets as commercial paper or other debt.e. These deposits are created when a check is drawn on a dollar deposit in a U.S. (A) is incorrect because Eurobonds are not always denominated in Eurodollars. Eurodollars are U. etc. interest ceilings. Answer (C) is correct. dollars on deposit in a foreign bank. Answer (B) is correct. they are negotiable).. the commercial paper of a top-rated (most creditworthy) company has the least risk. Examples of instruments traded in money markets are U. it can be sold only to sophisticated investors without registration.S. Hence.S. However. not lower. Answer (D) is incorrect because Eurobonds are usually issued not as registered bonds but as bearer bonds. usually long-term periods. Answer (C) is correct. commercial paper. CDs usually have a fairly high rate of return compared with other savings instruments because they are for fixed. money market mutual funds. Answer (A) is incorrect because negotiable CDs do have a secondary market (i. which usually consists of secured or unsecured promissory notes of large corporations. The absence of these costs means that Eurodollar deposit rates tend to be higher. Money market mutual funds invest in money market certificates such as treasury bills. commercial paper is a very short-term investment. Eurodollar market time deposits. A negotiable CD can be traded. Answer (A) is incorrect because a small firm may not have enough surplus cash to invest in commercial paper. monetary authorities.S. Also. transaction costs are lower. U. and commercial paper. Their advantage is that they are customarily less stringently regulated than most other bonds. The company discounts the time draft with the company's local bank and receives immediate payment. 40 . bank and then deposited in a bank outside the U. these mutual funds are superior to any single instrument. Answer (C) is incorrect because foreign bonds are denominated in the currency of the country in which they are sold. Answer (B) is incorrect because preferred shares are long-term capital market securities. A small firm with surplus cash should invest for the highest return and lowest risk. 41 .S. Because Eurodollars are outside the direct control of the U. and consumer credit loans. Commercial paper is preferable to stock or stock options because the latter represent only a residual equity in a corporation. dollars deposited outside the U. so names and nationalities of the investors are not recorded. negotiable CDs. Treasury bills. Answer (B) is incorrect because common stock does not have as high a priority in company assets as commercial paper or other debt. The ability to convert the investment into cash without a loss of principal is also important. banking regulations with respect to reserves. not higher. . and loans made thereon. Including the periods both before and after the end of the normal credit period. Those that are current (not past due) are listed in one column. money-market mutual funds with portfolios of short-term securities. Eurodollars are U. bankers' acceptances.S. Answer (C) is incorrect because the average collection period includes the total time before a payment is received. and the amount of time in one period does not necessarily bear any relationship to the other. It estimates the time between when the enterprise makes payments and when it receives cash inflows. monetary authorities. are tied to the LIBOR. For example.they are in banks outside the U. REQUIRED: The definition of “short-selling” DISCUSSION: (B) Short-selling is accomplished by borrowing securities from a broker and selling those securities. it is also called the receivables collection period. Answer (B) is incorrect because U. rates than larger deposits. Answer (D) is incorrect because the short seller is betting that the stock will decrease in price. 49 . Answer (A) is incorrect because Eurodollar borrowers tend to pay lower. Answer (B) is incorrect because it describes the concept of float. not smaller. Answer (A) is incorrect because the inventory conversion period (days of inventory) is the average time required to convert materials into finished goods and then to sell them. amounts. The seller speculates that the stock’s market price will decline. REQUIRED: The meaning of a firm’s average collection period. Answer (D) is incorrect because it describes the normal credit period. not higher. Because it is outside the direct control of the U. The theory is that the oldest receivables are the least likely to be collectible. net 30 should have an average collection period of somewhere between 15 and 30 days. dollars deposited in banks outside the U. Borrowers and depositors can both receive more favorable rates because. It is the average time required to convert the enterprise's receivables into cash. 45 .S. Answer (D) is correct. A convertible bond is not a short-term investment because its maturity date is usually more than one year in the future and its price can be influenced substantially by changes in interest rates or by changes in the investee's stock price. the Eurodollar market has lower costs. 50 .S. Eurodollars. Answer (B) is incorrect because Eurodollars are short-term marketable securities. banks. Treasury bills. Answer (C) is correct. REQUIRED: The item measured by an aging of accounts receivable. At a later time. 46 . the loan is repaid by buying securities on the open market and returning them to th broker.S. Furthermore. U. floating-rate preferred stock. Examples include U. those less than 30 days past due in another column.S. Marketable securities are near-cash items used primarily for short-term investment. plus the receivables collection period. A lower cost market can offer depositors higher interest rates.S. minus the payables deferral period (average time between resource purchases and payment of cash for them). Answer (A) is incorrect because margin trading involves buying securities bu borrowing money from a broker. reserve requirements and FDIC premium payments do not apply in this market. Answer (C) is incorrect because commercial paper is a short-term marketable security. It should be related to a firm’s credit terms. commercial paper. The day's sales outstanding (days of receivables) may be stated as the accounts receivable balance divided by average credit sales per day or as days in the year divided by the receivables turnover. Answer (B) is incorrect because the cash conversion cycle equals the inventory conversion period. This process typically occurs before the receivables collection period. DISCUSSION: (D) The purpose of an aging of receivables is to classify receivables by due date. with its lower costs. For example. rates. which is the rate paid on deposits of other large banks by the largest London banks with the highest credit standing.S. and negotiable CDs of U. Answer (A) is incorrect because U. Answer (D) is correct. a firm that allows terms of 2/15. dollars are on deposit in both cases. the Eurodollar market can offer smaller spreads between borrowing and lending rates. Answer (D) is incorrect because interest rates on these deposits.S. The amount in each category can then be multiplied by an estimated bad debt percentage that is based on a company’s credit experience and other factors. smaller deposits tend to earn lower. 47 . etc. Thus.S. 48 . Answer (D) is incorrect because the inventory divided by the sales per day is the inventory conversion period (days of inventory). Answer (C) is incorrect because the investor does not own the shares sold in a short sale. Treasury bills are short-term marketable securities. Answer (C) is incorrect because Eurodollar deposits tend to be for larger. DISCUSSION: (A) The average collection period measures the number of days between the date of sale and the date of collection. DISCUSSION: A firm’s average gross receivables balance can be calculated by multiplying average daily sales by the average collection period (days’ sales outstanding). Answer (A) is incorrect because tightening credit will reduce sales and bad debt losses. Increasing the quality of the accounts rejected means that fewer sales will be made. Thus. Answer (A) is incorrect because an aging schedule is used for receivables. not the credit term. Answer (C) is incorrect because an increase in cash sales with no diminution of credit sales will not affect receivables. Answer (B) is incorrect because an aging schedule concerns specific accounts. 54 . . A decline in the receivable turnover indicates that either sales are declining relative to receivables. Answer (D) is correct. Answer (B) is incorrect because small discounts although helpful. net 30. Answer (D) is correct. 52 . higher quality accounts will mean a shorter average collection period. Answer (B) is correct. Answer (A) is incorrect because sales are being collected more slowly when the turnover declines. the firm is collecting credit sales more slowly than before. 57 .the allowance) and will not affect the receivables balance and therefore the turnover ratio if an allowance system is used. Answer (D) is incorrect because a high turnover would be indicative of an efficient credit and collection department. Answer (A) is incorrect because write-offs do not reduce net receivables (gross receivables . sot the product could not be the dollar balance of receivables. Answer (D) is incorrect because inventories have no impact on the receivable turnover ratio.Aging the receivables and estimating the uncollectible amounts is one method of arriving at the appropriate balance sheet valuation of the accounts receivable account. would be unlikely to lead to an unusually high turnover. and thus is unrelated to credit and collection policy. or receivables are increasing relative to sales. and the effect of reducing the numerator and denominator by equal amounts is to increase the ratio if the fraction is greater than 1. A high receivables turnover is indicative of short credit terms and possibly the use of a lockbox system to speed up cash flows. most likely. REQUIRED: The calculation of the average gross receivable balance. The accounts receivable turnover ratio equals net credit sales divided by average receivables. Answer (D) is incorrect because the level of collection expenditures must be considered when implementing a collection policy. The marginal cost of a credit and collection policy should not exceed its revenue. not liabilities.10. Answer (C) is incorrect because. 53 . 56 . Answer (C) is correct. Answers (A) and (B) are incorrect because Alternative 1 cannot be correct. Answer (B) is correct. it will decrease if a company lengthens the credit period or the discount period because the denominator will increase as receivables are held for longer times. 51 . Answer (C) is incorrect because an increase in sales will increase the turnover ratio.0. 55 . The objective is to reduce bad debt losses and thereby maximize profits. the amount being calculated. Answer (C) is incorrect because an aging schedule focuses on uncollectible receivables. Answer (A) is incorrect because the quality of accounts is important to credit policy since it is inversely related to both sales and bad debts. REQUIRED: The reason for offering credit terms of 2. The company is therefore not trying to maximize its sales or increase its bad debt losses. A quantity discount is an attempt to increase sales by reducing the unit price on bulk purchases. Hence. Answer (C) is incorrect because offering a cash discount improves cash flow and reduces receivables and the cost of extending credit. Small discounts would not be as helpful as a lockbox system and short credit terms. Answer (B) is incorrect because a decline in sales near the end of the period signifies fewer credit sales and receivables. It concerns only the price term of an agreement. Alternatively. It contains average gross receivables. Answer (B) is incorrect because tightening credit will reduce sales and bad debt losses. Answer (A) is incorrect because a lenient credit policy would lead to a low turnover. annual credit sales can be divided by the accounts receivable turnover (net credit sales ÷ average accounts receivable) to obtain the average balance in receivables. not averages. Answer (D) is incorrect because Alternative III cannot be correct. Neither of the multiplicands is a dollar figure. the average collection period is a ratio calculated by dividing the number of days in a year (365) by the receivable turnover. Ryan must estimate the expected bad debt losses under each new policy. Answer (E) is incorrect because the bad debt loss percentage has apparently declined. Answer (D) is incorrect because no information is given relative to working capital elements other than receivables. 62 . Answer (B) is incorrect because the average collection period would decrease since the average time from cash disbursement to cash realization would be shorter. 60 . Answer (A) is incorrect because. the average collection period must have declined. 61 . REQUIRED: The true statement about a change in credit policy that has resulted in greater sales and a reduction in accounts receivable. DISCUSSION: (B) All factors should be considered that differ between the two policies. Answer (B) is incorrect because a decrease in the percentage discount offered provides no incentive for early payment. unless required to match competition. DISCUSSION: (B) The normal operating cycle is defined as the period from the acquisition of inventory to the collection of the account receivable. so an acceleration of customer payments will have no effect on working capital. Thus. Accomplishing both higher sales and a lower receivables increases the turnover and results in a shorter collection period. the higher the turnover. Accomplishing both higher sales and a lower receivables increases the turnover and results in a shorter collection period. . Answer (D) is incorrect because the discount may have been increased. REQUIRED: The true statement about extending credit for a period longer than the purchase’s operating cycle. Answer (A) is incorrect because a seller that extends long-term credit will have a higher level of receivables than a firm with a shorter credit period. Thus. regardless of its cost of capital. Answer (C) is incorrect because the seller is not guaranteed that a purchaser will resell the merchandise. 59 . Factors that do not differ. Answer (A) is incorrect because the operating cycle would decrease since the average time from cash disbursement to cash realization would be shorter. Answer (B) is incorrect because the ability to borrow at a lower rate is a reason for not offering cash discounts. Thus. credit terms are typically somewhat standardized within an industry. The average collection period is a ratio calculated by dividing the number of days in a year (365) by the receivable turnover. Accordingly. such as the current bad debt experience. The turnover increases when either sales (the numerator) increase or receivables (the denominator) decrease. Answer (D) is incorrect because more customers will take discounts. doubtful accounts are fewer. it would not offer cash discounts. a firm will likely be forced to match the competition or lose market share. Answer (C) is incorrect because accounts receivable turnover (sales ÷ average receivables) has increased. DISCUSSION: (A) An increase in discounts taken accompanied by declines in receivables balances and doubtful accounts all indicate that collections on the increased sales have been accelerated. Also. Answers (A) and (C) are incorrect because no statement can be made with respect to profits without knowing costs. a company would not offer them unless it desperately needed cash. are not relevant. which has led to quicker payments. or receivables (the denominator) decrease. Answer (C) is incorrect because the relationship between the cost of capital and the prime rate may not be relevant if the firm cannot borrow at the prime rate. Thus if most companies in the industry offer similar terms. REQUIRED: The factor not considered in an analysis of propose credit policies. REQUIRED: The true statement about a change in credit policy that has resulted in greater sales and a reduction in accounts receivable. if the company does not need cash. Answer (D) is incorrect because offering a discount may accelerate payment.DISCUSSION: (D) Because these terms involve an annual interest cost of over 36%. the shorter the average collection period. Accordingly. Both receivables and cash are elements of working capital. the shorter the average collection period. the seller must therefore be financing more than just the purchase of inventory. If trade credit is for a period longer than the normal operating cycle. If the cause of increased sales is an increase in the cash discount. the higher the turnover. 58 . Sales are up. cash would be collected more quickly than previously and the cash conversion cycle would be shortened. it can be inferred that the additional customers would pay during the discount period. DISCUSSION: (B) An increase in discounts taken accompanied by declines in receivables balances and doubtful accounts all indicate that collections on the increased sales have been accelerated. Answer (C) is correct. The turnover increases when either sales (the numerator) increase. or short-term. Also. Answer (A) is correct. The other statements are false. An increase in the proportion of short-term financing will not affect a company's degree of leverage. Factoring has been traditional in the textile industry for years. Ryan’s ability to increase receivables and possible bad debt losses may be limited. a firm increases its risk by financing with short-term debt because such debt must be “rolled over” frequently. Answer (B) is incorrect because inventory should be increased to accommodate higher sales levels. plus a fee for collection. 67 . and recently companies in many industries have found it an efficient means of operation. 68 .and short-term loans result in liquid assets. and bad debts. 64 . The financing cost is usually high: about 2 points or more above prime. the secured party still may proceed against the borrowers.Answer (A) is incorrect because the cost of funds is an obvious element in the analysis of any investment. Answer (D) is incorrect because factoring is a source of short-term funds through sale of receivables. For example. perhaps before expected funds from a new project. REQUIRED: The item that provides a spontaneous source of financing. and usually lend some amount less than the face value of the receivables. a party that has agreed to deliver a commodity at a certain date may enter into a hedging contract to buy an equal quantity on the same date. Answers (B) and (D) are incorrect because mortgage bonds and debentures do not arise automatically as a result of a purchase transaction. Pledging accounts receivable is an arrangement in which receivables are used as security for a loan. retain the right of recourse against the borrower in case of default. It is irrelevant whether the borrowed funds are long. Answer (C) is incorrect because the impact on the current customer base of extending terms to only certain customers is relevant. Lenders in these circumstances choose the collateral with care. Answer (C) is incorrect because factoring is a source of short-term funds through sale of receivables. DISCUSSION: (A) Trade credit is a spontaneous source of financing because it arises automatically as part of a purchase transaction. 63 . accounts receivable staff. 66 . which can often operate more efficiently than its clients because of the specialized nature of its service. Answer (B) is incorrect because leverage is the use of borrowed funds to earn returns for stockholders. which are expected to rise. A factor purchases a company's accounts receivable and assumes the risk of collection. Both long. Answer (C) is incorrect because the length of a loan does not affect the amount of liquid assets. . Any loss on one transaction should then be offset by the gain on the other. The seller receives money immediately to reinvest in new inventories. purchase orders will increase to accommodate the higher sales levels. trade credit is the largest source of short-term financing for may firms both large and small. As sales increase. 65 . the danger of default is increased. But if the receivables are not paid. Answer (B) is correct. Answer (D) is incorrect because a just-in-time system is not used when a company orders inventory once a quarter. Answer (B) is incorrect because factoring is a source of short-term funds through sale of receivables. Consequently. Answer (A) is incorrect because factoring is the outright sale of receivables for cash at a discount. Relaxing the credit policy for customers will lead to increased sales because more people will be eligible for more credit. Answer (A) is correct. Answer (A) is correct. future interest rates are difficult to predict. Answer (C) is incorrect because safety stock is based on expected sales. Because the debtor company will be forced to meet principal and interest payments quickly. However. These reductions in costs can more than offset the fee charged by the factor. Because of its ease in use. Answer (D) is incorrect because existing loan agreements may require Ryan to maintain certain ratios at stated levels. The current customers may demand the same terms. Answer (C) is incorrect because the use of receivables as a financing source requires an extensive factoring arrangement and often involves the creditor’s evaluation of the credit ratings of the borrower’s customers. Answer (D) is incorrect because hedging is the process of protecting oneself against loss because of future price changes. A company that uses a factor can eliminate its credit department. thus. Under normal conditions the yield curve is upward sloping. Answer (C) is incorrect because receivables may be sold with or without recourse. short-term interest rates are lower than long-term interest rates. Answer (D) is incorrect because an increase in current liabilities decreases the current ratio. but risk is increased because of the need for frequent refinancing. a firm financing with short-term debt will pay less interest than a firm financing with long-term debt--increasing its ROE. and the firm is exposed to the volatility of short-term rates. Thus. Answer (B) is incorrect because the denominator of the first term should represent the funds made available by not taking the discount (100 . 75 . outside of owner financing. such as small businesses but especially for buyers. Answer (D) is incorrect because mortgage bonds are a long-term source of financing. Answer (B) is incorrect because the buyer is paying the amount of discount not taken in exchange for the extra 45 days of credit. Trade credit is an important source of credit for all businesses but especially for buyers. which is higher than the prime rate has ever been. Answer (C) is incorrect because prepaid interest is not a source of financing. 73 . trade credit is subject to the risk of buyer default. the result is that the cost of trade credit. 71 . DISCUSSION: (D) Payments should be made within the discount periods if the cost of not taking discounts exceed the firm’s cost of capital. Answer (A) is incorrect because trade credit is an important source of financing for small firms.discount %). Answer (C) is incorrect because paying 2% for 20 days of credit is more expensive than paying 2% for 50 days of the same amount of credit. If the discount period is longer. net 30 exceeds 36% annually. For example. REQUIRED: The true statement about cash discounts. Answer (C) is incorrect because percentage financing cost is unaffected by the purchase price of the items. A precise formula would incorporate the effects of compounding when calculating the annual cost. Answer (B) is incorrect because it occurs as a result of transactions apart from purchase transactions. Answer (A) is incorrect because installment loans are usually a longer-term source of financing and are more difficult to acquire than trade credit. Answer (B) is incorrect because commercial paper is normally used only by large companies with high credit ratings. the cost of not taking the discount. Because of its ease in use. Answer (D) is incorrect because the cost of trade credit depends on the credit terms and the price paid. The multiple of these terms is the approximate annual percentage cost of not taking the trade discount. that is. Answer (B) is incorrect because trade credit is ordinarily short-term source of financing. DISCUSSION: (C) Trade credit is a spontaneous source of financing because it arises automatically as part of a purchase transaction. 70 . In other words. The terms of payment are set by the supplier. The second term represents the number of times per year this cost is incurred. A seller with generous payment terms may charge a higher price for its merchandise. Thus the cost of not taking the discount is usually higher than the cost of a bank loan. such as small businesses. REQUIRED: The true statement about trade credit. Assuming other factors are constant. failing to take a discount when terms are 2/10. the only options available. In other words. trade credit is the largest source of short-term financing for many firms both large and small. net 30 means that the firm is paying an effective annual interest rate exceeding 36%.69 . are bank loans and a line of credit from suppliers. Answer (C) is correct. It is this latter . Like all forms of financing. but trade credit usually requires payment within a short period of time. In fact. Answer (C) is correct. such credit is arranged separately from the transactions to acquire the assets being financed. A small retail store would not have access to major capital markets. Answer (A) is correct. the days of extra credit obtained by forgoing the discount are fewer. Answer (B) is incorrect because percentage financing cost is unaffected by the purchase price of the items. Answer (C) is incorrect because the first term is the reciprocal of the correct term. Answer (A) is incorrect because it occurs as a result of transactions apart from purchase transactions. Answer (D) is correct. is greater. the lower the opportunity cost of forgoing the discount and using the trade credit financing. such credit is arranged separately from the transactions to acquire the assets being financed. calculated as the cost per unit of trade credit (discount %) divided by the funds made available by not taking the discount (100 discount %). Answer (A) is incorrect because the cost of not taking a discount when terms are 2/10. 74 . Answer (D) is correct. Answer (D) is incorrect because the second term is the reciprocal of the correct term. that might not have access to other credit markets. 72 . Trade credit is a spontaneous source of financing because it arises automatically as part of a purchase transaction. The first term of the formula represents the periodic cost of the trade discount. Trade credit is a spontaneous source of financing because it arises automatically as part of the purchase transaction. Answer (A) is incorrect because the lower the discount percentage. One reason is that less risk is involved in the short run. Future inflation is incorporated into this relationship. Inc. The Shad loan would be for $97 at a cost of $3. Answer (C) is incorrect because the prime rate is a bank loan rate. not paying Shad. DISCUSSION: (A) Historically. This is lower than the rate on the bank loan (17. For this reason. Answer (C) is incorrect because short-term rates are more likely to be greater than long-term rates if current levels of inflation are high.'s invoices on time would be the lowest cost source of capital. downward if interest rates are anticipated to fall. Answer (D) is incorrect because long-term rates may be viewed as short-term rates adjusted by a risk factor. The loan would be for 75 days (90 .302% because it is a discount loan). Thus. 78 . Assuming a $100 invoice.302% ($14. These compensating balances effectively increase the rate of interest on the money borrowed from the bank. Answer (B) is incorrect because short-term rates are usually lower than long-term rates.25). it is often required to keep a certain percentage of the funds in the bank at all times. by not paying on the 15th day. The annual interest rate is 18.15). When a firm borrows money from the bank. whereas business debtors often prefer to pay higher rates on long-term debt to avoid the hazards of short-term maturities. Answer (A) is incorrect because the actual annual percentage rate based on forgoing Shad's discount is 14. 80 . Another consideration is liquidity preference. giving an interest rate of 17. in recent years. or the cost of forgoing the discounts allowed by Fort Co. one facet of the term structure of interest rates (the relationship of yield and time to maturity) is that short-term interest rates have ordinarily been lower than long-term rates.182%) and Riley Co. (18.326%). the company will have the use of $98 for 45 days (60-day credit period . Most economists believe that a long-term interest rate is an average of future expected short-term interest rates. not a conservative one. Moreover. Answer (B) is incorrect because the effective rate on most companies' bank loans will be much higher than the prime rate. It is traditionally the lowest rate charged by banks.845% ($3/$97 x 4. (16.8 interest periods in a year (360/75).326%). Thus. at a cost of 14.75/$85.75% on a discount basis. REQUIRED: The true statement about short-term interest rates. On a $100 note. banks have been making loans at still lower rates in response to competition from the commercial paper market.845%. Answer (B) is incorrect because a retail store must have instant access to its inventory to provide continuous services to customers.1818% [(360/20) periods x ($1/$99)]. 79 . The interest would be 16.alternative that is most often used because it permits the store to finance inventories for 30 to 60 days without incurring interest cost. Given 4. The Riley Company discount represents an interest charge of $2 on a loan of $98. i. It is not well-suited to financing inventory of a small retailer with high turnover because of the difficulty of identification. However.326% ($2/$98 x 8 periods).8). the annual interest rate would be 14. The prime interest rate is the rate charged by commercial banks to their best (the largest and financially strongest) business customers. Answer (A) is incorrect because the prime rate has nothing to do with a commitment fee on a bank loan. the yield curve will slope upward if future rates are expected to rise. Answer (D) is correct. a terminal warehouse receipt loan would not be suitable because the inventory would not be in the immediate possession of the seller. or the cost of forgoing the discounts allowed by Fort Co.845%. (18. the borrower would only receive $85.10-day discount period). This is lower than the rate on the bank loan (17. the Fort Company discount represents interest of $1 on a loan of $99 for 20 days (30-day credit period .182%) and Riley Co. and remain flat if investors think the rate is stable. future expectations concerning interest rates affect the term structure. (16. Answer (B) is incorrect because the actual annual percentage rate based on forgoing Shad's discount is 14. The number of periods in a year would be 8 (360/45). 77 . Answer (D) is correct. not the rate on commercial paper.302% because it is a discount loan). (18.182%) and Riley Co.845%. . (16.. The first step is to determine the actual annual percentage interest rate for each of the four options.845%.25. or the cost of forgoing the discounts allowed by Fort Co. Investors in an uncertain world will accept lower rates on short-term investments because of their greater liquidity.e. Statement b illustrates an aggressive financing policy.15-day discount period).302% because it is a discount loan). The bank loan was quoted at 14. A line of credit is an arrangement between a bank and a borrower in which the bank commits itself to lend up to a certain maximum amount to the borrower in a given period. Answer (D) is incorrect because a chattel mortgage is most often used for financing movable equipment. This is lower than the rate on the bank loan (17. Answer (C) is correct. Answer (A) is incorrect because only large companies with excellent credit ratings have access to the commercial paper market. 76 . Answer (C) is incorrect because the actual annual percentage rate based on forgoing Shad's discount is 14.326%). interest Answer (A) is incorrect because it is the formula for the simple interest rate for a 1-year loan. lines of credit. however. Answer (B) is incorrect because it is the formula for the add-on installment interest for a 1-year loan. it calls for the buyer to pay a specified amount. not the bank. Revolving credit. An unsecured loan is a loan made by a bank based on credit information about the borrower and the ability of the borrower to repay the obligation. the full $20. Answer (C) is incorrect because interest charged on a discount basis results in a deduction from the face value. in financial accounting. Answer (C) is incorrect because a cash balance held in reserve for random. a valuation allowance is used to reflect losses on marketable securities. Answer (D) is incorrect because. The borrower does not receive the full face value of the loan. Answer (D) is incorrect because interest is charged on an add-on basis when the face value of the loan initially equals the borrowed amount plus the nominal interest charge. for a fee. Answer (D) is incorrect because large depositors may receive favorable treatment. This requirement raises the real rate of interest to the borrower. Answer (C) is incorrect because a safety stock of inventory is held to avoid inventory stockouts. Banks sometimes require a borrower to keep a certain percentage of the face amount of a loan in a non-interest-bearing checking account. Answer (D) is incorrect because the description is of a conditional sales contract except that the seller. 81 . Answer (B) is incorrect because an open account is a credit arrangement involving only the signing of an invoice by the buyer. retains title to the goods until the buyer has completed payment. and commercial paper are all unsecured means of borrowing. the seller's bank may. that is. 83 . 84 . A time draft (trade acceptance) is a form of commercial draft because it is drawn by a seller on the buyer. Unsecured credit is not backed by collateral. guarantee payment. Answer (B) is correct. which is characteristic of a sight draft. This greater rate compensates a bank for services provided and results in greater profitability for the financial institution. if a firm chooses not to use its full line of credit and is not charged for the unused portion. Simple interest is charged on the amount actually paid to the borrower. 85 . The discounted interest rate is based on the amount borrowed but is paid in advance. If interest is charged on a simple basis. Answer (A) is incorrect because a sight draft calls for immediate payment upon delivery of the shipping documents to. bankers' acceptances. which transmits the draft to the buyer. Answer (C) is correct. A time draft. Answer (D) is correct. Answer (B) is correct. Answer (A) is incorrect because the cash balance necessary for a firm to conduct day-to-day business is a transactions balance. A bank may require a borrower to keep a certain percentage of the face value of a loan in the firm's account. The draft and the shipping documents related to the goods are then sent to the buyer's bank. Answer (B) is incorrect because a restriction on a new issuance does not raise the interest rate on money previously borrowed. Answer (A) is correct. Answer (A) is incorrect because interest is charged on a discount basis when it is deducted from the face value borrowed.000 face value of the loan is made available to the borrower. It is calculated using the following formula: Interest Borrowed amount . This requirement raises the effective rate of interest paid by the borrower. It should float up or down with the prime rate. 82 . Answer (D) is incorrect because a cash balance that is held to enable the firm to take advantage of any bargain purchases that might arise is a speculative balance. The loan is not secured by collateral. A minimum checking account balance that a firm must maintain with a commercial bank is a compensating balance. Answer (B) is incorrect because. This banker's acceptance is an assumption of the obligation to pay at the due date. The buyer accepts the draft by signing it. but compensating balances are funds maintained by loan recipients for the benefit of the lender. Answer (D) is incorrect because the correct formula for the discounted interest rate is interest divided by the borrowed amount less interest. If a seller is reluctant to ship goods because of concern about the buyer's ability to pay a time draft. Answer (C) is correct. and the acceptance of the draft by. 86 . . but is made on the signature of the borrower. unforeseen fluctuations in cash inflows and outflows is a precautionary balance. the buyer. is similar to a promissory note because it is payable at a specific time in the future rather than upon acceptance by the buyer. the rate of interest on the portion used does not increase.Answer (A) is incorrect because the floating interest rate is not always higher. Commercial paper is the term for the short-term (typically less than 9 months). Answer (B) is incorrect because commercial paper is often issued directly by the borrowing firm. Among the largest issuers of agency securities (excluding the Treasury) are the Federal Home Loan Banks. Maturities usually do not exceed 270 days. but that "most dealers or organizations will repurchase an issue that they have sold. avoidance of compensating balance requirements. c. a floating lien is secured by property. and the other entities that provide credit to farmers and home buyers. Commercial paper is a form of unsecured note that is sold by only the most creditworthy companies. Answer (D) is incorrect because investors must pay a commission similar to that on other investment securities. SMA 4M. Commercial paper is a type of unsecured promissory note issued by large. Statements b. and costly financing arrangements are avoided. the Federal National Mortgage Association (Fannie Mae). Answer (B) is incorrect because bankers' acceptances are issued by commercial banks to finance specific transactions. a floating lien is secured by property. observes that no general (active) secondary market exists for commercial paper. The market is not open to all companies because only major corporations with high credit ratings can participate. Answer (D) is incorrect because interest rates must be higher than those of Treasury bills to entice investors. Answer (C) is correct. Commercial paper provides a broad and efficient distribution of debt. The acceptance by the bank guarantees payment at maturity. It is issued at a discount from its face value and has a maturity period of less than 270 days. Answer (A) is incorrect because an agency security is issued by a corporation or agency created by the U. Answer (A) is correct. Commercial paper usually carries a low interest rate in comparison to other means of financing. and no compensating balances are required. Answer (A) is correct. Commercial paper is more risky than Treasury bills. They are normally used to finance a specific transaction. Answer (B) is incorrect because a chattel mortgage is a loan secured by personal property (movable property such as equipment or livestock). such as the Federal Housing Administration. Understanding Financial Instruments. the composition of which may be constantly changing. such as inventory. Statement a is incorrect. such as inventory.S. d. unsecured.Answer (A) is incorrect because a chattel mortgage is a loan secured by personal property (movable property such as equipment or livestock). the maturity date is only a few days after issuance. and broad debt distribution are advantages of commercial paper." Answer (A) is incorrect because commercial paper usually has a maturity date of 270 days or less to avoid securities registration requirements. such as the Federal National Mortgage Association (Fannie Mae). Commercial paper usually carries a low interest rate in comparison to other means of financing. Answer (C) is incorrect because a chattel mortgage is a loan secured by personal property (movable property such as equipment or livestock). is an agency security (agency issue). strong firms. unsecured. Answer (B) is incorrect because only large companies with good credit ratings can find buyers for their commercial paper. It allows the buyer to retain interest income although the seller-dealer can repurchase after a specified time. unsecured note payable issued in large denominations by major companies with excellent credit ratings. 92 . Answer (C) is correct. Commercial paper is a lower cost source of funds than bank loans. Answer (D) is incorrect because a repurchase agreement involves what is in essence a . Answer (B) is incorrect because bankers' acceptances are drafts drawn on deposits at a bank. Commercial paper is a form of unsecured note that is sold by only the most creditworthy companies. promissory note issued by a commercial enterprise. 89 . large denomination (often over $100. Answers (A). and therefore the right answer. DISCUSSION: (C) Commercial payer is a short-term. 91 . the composition of which may be constantly changing. A short-term security issued by a corporation or agency created by the U. creditworthy companies to other companies and institutional investors. Answer (C) is incorrect because commercial paper is unsecured. In many instances. government. Answer (C) is incorrect because commercial paper is a short-term. Other issuers of home mortgage-backed securities include the Government National Mortgage Association (Ginnie Mae) and the Federal Home Loan Mortgage Corporation (Freddie Mac).000) promissory notes issued by large. 90 . Also. Also. (B) and (D) are incorrect because lower rates.S. 88 . 87 . Answer (D) is incorrect because a repurchase agreement involves a secured loan to a government securities dealer. It is issued at a discount from its face value and has a maturity period of 270 days or less. government. and e are all accurate statements. REQUIRED: The item not an advantage of using commercial paper for short-term financing. Factoring is a form of financing in which receivables serve as security. Examples are government securities issued by the bodies that finance mortgages. 97 . the least frequent compounding is associated with the lowest effective annual percentage cost.1. Annual compounding is less frequent than semiannual.000 for the 20 days. Answer (D) is incorrect because a line of credit is an arrangement. The higher the compensating balance. and the effective cost is higher. Answer (B) is correct. insurance companies. quarterly. 96 . quarterly. and monthly compounding are all more . Also.0 = 19. and monthly compounding are all more frequent than annual compounding. quarterly. A compensating balance results in a higher effective borrowing cost because the compensating balance is an amount of cash that the firm is unable to use.000 in interest to hold the $99. The corporation can obtain trade credit for 20 additional days by not paying within the discount period. of 21. The term of the loan is not relevant to the calculation of the effective annual percentage cost of financing.20/12)]12 . Instead of paying $99. discount interest is deducted from the loan funds in advance. It represents ownership of the goods and is ordinarily needed to obtain the goods from the bailee.1.. the higher the portion of the loan funds that must be left on deposit with the lender. Discount interest results in a higher effective borrowing cost than simple interest because the bank deducts interest in advance so the borrower receives less than the face value of the loan.secured loan to a dealer in government securities. or monthly. Answer (A) is correct. Answer (C) is incorrect because lower compensating balances and regular interest are less costly. The following is the calculation: Effective rate = [1 + (.94% Therefore. Security for short-term inventory financing can be arranged if the debtor places its inventory under the control of the lender or its agent (e.g. the 30-day note has an effective annual rate of 21. if compounding effects are considered. Answer (B) is correct. of 21. Answer (D) is incorrect because the note has an effective rate. a public warehouse). 94 . A warehouse receipt is issued by a person engaged in the business of storing goods for hire. Answer (C) is incorrect because a compensating balance is disadvantageous to the borrower.000/$99. The following is the calculation: Effective rate = [1 + (. the more frequent the interest compounding.1. the more frequent the interest compounding. short-term promissory note issued by large firms to other firms. Answer (D) is correct.18% [($1. The cheapest terms. is found using the following formula: Effective rate = [1 + (1.83% depending on the method of calculation.94%. Answer (A) is incorrect because commercial paper is a type of unsecured. etc. mutual funds.0 = 21. which may be formal or informal. Answer (C) is incorrect because a revolving credit agreement is a formal line of credit.20/12)]12.94%. The most costly combination of characteristics is a higher compensating balance and discount interest.94%. given that all options have the same nominal interest rate. Answer (A) is incorrect because the effective trade credit rate is 19. and monthly compounding are all more frequent than annual compounding. Answer (C) is incorrect because the note has an effective rate. The two major types of documents of title are bills of lading (issued by carriers) and warehouse receipts.94%. Answer (D) is incorrect because discount interest and a compensating balance are disadvantageous to the borrower.000 at the end of 30 days. Hence. Answer (B) is incorrect because lower compensating balances and regular interest are less costly. Because a 360-day year has 18 such periods. usually with a bank. and the lender holds the warehouse receipts. Answer (B) is incorrect because. quarterly. the more costly the loan.0 = 21. including compounding effects. Answer (D) is incorrect because. resulting in a further increase in the effective financing cost.83%. In comparison.000 to satisfy its obligation within 10 days. will be simple interest with no compensating balance. 93 . it can pay $100.94%. the corporation should use trade credit to obtain the short-term credit. A document of title is usually issued by a bailee covering goods in the bailee's possession or care (UCC 1-201). The most desirable set of terms are those that result in the lowest cost of borrowing.0 = 21.000/99. Answer (C) is incorrect because. the more costly the loan.000)]18. Semiannual. the rate is higher. the interest rate is approximately 18.1. including compounding effects. The effective rate. Answer (A) is incorrect because lower compensating balances and regular interest are less costly. the interest paid is charged on a smaller amount of funds available to be used by the borrower. 95 . taking compounding into consideration. calculated as follows: [1 + (. between a commercial bank and its customer concerning the maximum loan amount available. For any given quoted nominal rate. Semiannual. that large firms often use. the more frequent the interest compounding.20/12)]12. The corporation will thus incur $1. Answer (B) is incorrect because discount interest is disadvantageous to the borrower. the more costly the loan. Semiannual. Answer (A) is correct. However.000) x 18]. Answer (D) is correct.000 $20.000 unused portion. it will pay 17% interest on $80.000 discount) and 1% on the $20. The effective interest rate would thus be 17. its effective rate is 25% ($20. . a total of $13.000).800 ÷ $80.000 ($100. Answer (C) is incorrect because the effective rate would be 18. If the company chooses the line of credit.25% ($13.frequent than annual compounding. 98 .000 available funds].000) ÷ $80.800. Answer (A) is incorrect because if the company forgoes the cash discount.000 ÷ $80. Answer (B) is incorrect because an 18% effective rate exceeds that on the line of credit.75% [(15% x $100.000 immediate cash price).
Copyright © 2024 DOKUMEN.SITE Inc.